You are on page 1of 84

Technology, R&D, and Efficiency

Macro – Answers to Review Questions

B. Answers to Short-Answer, Essays, and Problems, chapter 1

3. What do economists mean when they say that “there is no free lunch”? Give another
example to which this statement applies.

Anything of any value that is offered for “free” still has a cost. Economists refer to this
sacrifice as an opportunity cost. In this case, the resources that were used to provide the
free lunch could have been put to an alternative use. The opportunity cost is the next
best alternative use for those resources. As another example, consider the case of a bank
that offers you a “free” sports bag to open an account at the bank. The bag may be free
to you as a new bank customer, but there is still a cost paid by the bank in the form of
resources that could have been put to alternative uses. [text: E pp. 3-4; MA pp. 3-4; MI
pp. 3-4]

9. Explain the importance of the ceteris paribus or “other-things-equal” assumption.

Because economics is concerned with real-world behavior, it is impossible to develop


theories about economic relationships in a laboratory setting where the variables of
interest could be isolated. Economists try to analyze changes in the variables of interest
by finding ways to hold “other things constant or equal.”

Thus, the ceteris paribus assumption is made to indicate that these other variables are
not changing or affecting the variables of interest. For example, the theory of consumer
demand states that price and quantity demanded are inversely related; people will buy
less at higher prices than they will at lower prices. But this theory assumes that other
variables that might affect quantity demanded are not changing. This assumption is the
ceteris paribus assumption. [text: E pp. 7-8; MA pp. 7-8; MI pp. 7-8]

New 13. What is policy economics? What are the three basic steps on policymaking?

Policy economics applies economic facts and principles to help resolve specific
problems and to achieve certain economic goals. The three basic steps on formulating
economic policy are: (1) state the goals; (2) determine the policy options to be used to
achieve the stated goals; and (3) implement and evaluate the options on the basis of
specific criteria important to decision makers. [text: E p. 8; MA p. 8; MI p. 8]

14. List eight widely accepted economic goals of the United States.

Eight goals are given in the text: economic growth, full employment, economic
efficiency, price stability, economic freedom, equitable distribution of income,
economic security, and balance of international trade. [text: E p. 9; MA p. 9; MI p. 9]

17. Below are six statements. Indicate whether each one pertains to microeconomics (MIC)
or macroeconomics (MAC).

(a) “The inflation rate in the United States hit its lowest level in the last twenty years.”
(b) “The profits of Microsoft rose 20 percent during the past quarter.”
(c) “Rains from El Nino again hit the California region causing severe flooding in
farms. The prices for citrus and produce are expected to rise sharply.”
(d) “The nation’s economy grew at an annual rate of 3.7 percent in the final quarter of
the year.”
(e) “The trade deficit in the United States was $20 billion last month.”
(f) “General Motors plans to spend $800 million on a new automobile plant.”

153
Technology, R&D, and Efficiency

(a), (d), and (e) are macro; (b), (c), and (f) are micro. [text: E pp. 9-10; MA pp. 9-10;
MI pp. 9-10]

19. Below are six statements. Identify whether each is a positive or normative statement.

(a) The minimum wage would be increased so low-income workers can earn a living
wage.
(b) The unemployment rate is too high and should be reduced through government
actions.
(c) The rate of inflation was about 2 percent last year, an all time low for the past
decade.
(d) The government should take action to break up the monopoly power of Microsoft.
(e) Interest rates should be lower in the United States so that people can afford to build
a home.
(f) The Federal government achieved a budget surplus for the first time in thirty years.
(a), (b), (d) and (e) are normative; and (c) and (f) are positive. [text: E p. 10; MA p. 10;
MI p.10]

23. Below are four statements. Each of them is an example of one of the pitfalls often
encountered in the study of economics. Indicate following each statement the type of
pitfall involved.

(a) “July is the month with the most ice cream sales and also the month with the most
drownings. Therefore, the more ice cream people eat, the more likely they are to
drown.”
(b) “Dry weather in the county where Farmer Brown lives decreased his income
because his crop was so poor. Therefore, when there is dry weather in the nation as a
whole all farm incomes will suffer.”
(c) “I have to live within my income. Therefore, governments should not be allowed to
borrow money.”
(d) “National health insurance plans are socialistic.”
(a) Causation is confused with correlation. (“Post hoc” fallacy.)
(b) This is the fallacy of composition. What is bad for one farmer is not necessarily bad
for all farmers if prices rise enough to offset the decline in crop yields overall. However,
dry weather in only one county would not cause an increase in agricultural prices, so
Farmer Brown would suffer if his were the only dry area.
(c) This illustrates two pitfalls. The fallacy of composition may be a factor behind this
statement since governments are a collection of individuals, but the fallacy is that
governments do not have limited life spans and additionally have the power to tax.
This statement also illustrates biased thinking since it assumes that all borrowing is bad.
(d) This is an example of loaded terminology designed to influence one’s view of
national health insurance plans. [text: E pp. 10-12; MA pp. 10-12; MI pp. 10-12]

New 24. What is the fallacy of composition? Give an economic and a non-economic example.

It is the incorrect reasoning that what is true for an individual (or part of a group) is
necessarily true for the whole group. Or, what is true at the micro level of analysis may
not be true at the macro level of analysis. Economic example: when an individual
farmer produces a large crop, then the farmer should have an increased income because
he or she has more output to sell. If, however, all farmers produce more output, then the
increase in output may decrease prices and reduce farm income. Non-economic
example: If a spectator at a packed basketball arena stands up, then he or she will likely
see the game better. If, however, all spectators at the game stand up, then the group of
spectators as a whole will not be able to see the game better. [text: E p. 11; MA p. 11;
MI p. 11]

154
Technology, R&D, and Efficiency

New 25. Explain what the post hoc fallacy is. Give an example.

It means “after this, therefore because of this.” It is the mistaken belief that when one
event precedes another, the first event is the cause of the second. An example: I
washed my car today; therefore it will rain tomorrow. [text: E pp. 11-12; MA pp. 11-12;
MI pp. 11-12]

26. Explain the difference between correlation and causation and give an example.

Correlation refers to a systematic and dependable association between two sets of data
(two kinds of outcomes). Causation implies that there is a cause-effect relationship
between two events. Correlation does not imply causation. Just because two events are
related in a predictable manner does not necessarily mean that one causes the other.
More must be known about the cause-effect relationship before conclusions about
causation can be drawn.

For example, one could discover a positive correlation between ice-cream sales and the
number of drownings. However, this does not mean that eating ice cream causes
drowning, nor does it mean that more drownings cause people to buy ice cream! [text:
E p. 12; MA p. 12; MI p. 12]

D. Answers to Appendix Questions

31. Define what is meant by a positive or direct relationship between two variables and
describe the line graph depicting such a relationship.

A positive or direct relationship between two variables describes a situation where the
two variables change in the same direction. If the first variable increases, the second
variable increases; if the first decreases, the second decreases. An example would be
individual income and spending. Generally, high spending is associated with high
incomes and lower spending is associated with lower incomes. The line graph of a
direct, positive relationship is upward sloping from left to right. [text: E pp. 15-16; MA
pp. 15-16; MI pp. 15-16]

32. Define what is meant by an inverse relationship between two variables and describe the
line graph depicting such a relationship.

An inverse relationship describes a situation where the two variables change in opposite
directions. When the first variable increases, the second variable decreases and vice
versa. An example would be product price and quantity demanded of the product.
Other things being equal, the higher the product price, the less will be purchased. The
line graph of an inverse relationship has a negative slope; that is, it is downward sloping
from left to right. [text: E pp. 15-16; MA pp. 15-16; MI pp. 15-16]

33. Differentiate between the independent and dependent variables in an economic


relationship.

The dependent variable changes as a consequence of the change in the independent


variable. By specifying one variable as the dependent variable, a causal relationship is
implied with changes in the independent variable causing changes in the dependent
variable. The dependent variable is the “effect” or outcome. [text: E pp. 16-17; MA;
pp. 16-17; MI pp. 16-17]

The slope at point A is 5/5 = 1, at B it is zero, at C it is -5/5 = -1. To find these slopes
divide the vertical distance of the tangent by the horizontal distance. At point C the line

155
Technology, R&D, and Efficiency

slopes down, so the vertical distance is negative when the horizontal direction is
positive. [text: E p. 19; MA p. 19; MI p. 19]

B. Answers to Short-Answer, Essays, and Problems, chapter 2

2. List the four resource categories and give a brief description of each.

(a) Land: natural resources including land, forests, water and minerals.
(b) Capital: investment goods or those manufactured items used in production of other
goods. Factories, tools, machinery, transportation facilities, and equipment are
examples. Money is not a capital good.
(c) Labor: a broad term used to describe the physical and mental talents of men and
women available to be used in producing goods and services.
(d) Entrepreneurial ability: a type of human resource, but unique from productive labor
in that it refers to the person who is the driving force behind production decisions,
innovation, and the one who is willing to take the risk of time, effort, reputation, and/or
funds. [text: E p. 23; MA p. 23; MI p. 23]

5. Explain and evaluate: “If resources were infinitely abundant in relation to the demand
for them, the economizing problem would dissolve in a sea of affluence.”

If this were true, people would not have to make choices and there would be no need for
economic systems to distribute the goods and services produced. In a world of
abundance, people could simply help themselves to whatever they wanted. [text: E pp.
22-24; MA pp. 22-24; MI pp. 22-24]

9. Differentiate between allocative efficiency and productive efficiency.

Allocative efficiency means that resources are used for the goods and services most
wanted by society. For example, if society wants compact discs instead of cassette
tapes, then compact discs should be produced with resources rather than cassette tapes.
Productive efficiency means that the least costly production techniques are used once
decisions are made about what will be produced. [text: E pp. 24-25; MA pp. 24-25; MI
pp. 24-25]

New 12. The production possibilities curve below show the hypothetical relationship between the
production of food and clothing in an economy.

(a) What is the marginal opportunity cost of producing the second unit of clothing?
(b) What is the total opportunity cost of producing the second unit of clothing?
(c) What is the marginal opportunity cost of producing the third unit of clothing?
(d) What is the total opportunity cost of producing the third unit of clothing?

Combination Food Clothing


A 0 4
B 7 3
C 13 2
D 18 1
E 22 0

(a) 5 units of food (18 – 13 = 5); (b) 9 units of food (22 – 13 = 9); (c) 6 units of food
(13 – 7 = 6); (d) 15 units of food (22 – 7 = 15). [text: E: pp. 25-27; MA: pp. 25-27; MI:
pp. 25-27]

15. What is the economic rationale for the law of increasing costs?

156
Technology, R&D, and Efficiency

Economic resources are not completely adaptable to alternative uses. In a two-product


(A and B) economy, an increase in the production of product A will cause a reduction in
the quantity of product B that can be produced because resources are being reallocated
from the production of B to A. That reallocation of resources is not constant and
becomes increasingly costly in terms of the lost production of B. As more resources
shift from the production of B to A, these resources are less and less adaptable or
suitable for the production of A. The production of more and more of A entails an
increasing opportunity cost in the form of less and less production of B. [text: E p. 27;
MA p. 27; MI p. 27]

18. In the following graph, explain the relationship between marginal cost and marginal
benefit at 1 million units of output, 2 million units of output, and 3 million units of
output for the production of computers. In your explanation discuss the overallocation
of resources, underallocation of resources, and optimal allocation of resources for the
production of computers.

At 1 million units of output, the marginal cost is $4 and the marginal benefit is $12.
There is underallocation of resources to computer production. For each additional
computer produced up to 2 million, the marginal benefits are greater than the marginal
costs. At 2 million units of output, the marginal cost is $8 and the marginal benefit is
$8. This point represents allocative efficiency and would be the optimal level of output.
The benefits of an additional unit of output just equal the additional cost at this level of
production. At 3 million units of output, the marginal cost is $12 and the marginal
benefit is $4. There is an overallocation of resources to computer production at this
level of output. Society would be better off if there was less output. In fact, production
of each unit after 2 million has a marginal cost that is greater than the marginal benefit.
[text: E p. 28; MA p. 28; MI p. 28]

22. What do economists mean when they state that investment is spending on “goods for the
future”?

It is easiest to explain this statement by realizing that consumption is spending on goods


and services for the present. The resources that are not used for consumption can be
devoted to goods or services that can be used in future production. For example,
investment in education, machinery, factories, equipment, and tools are types of
spending that will be used in expanding production of consumer goods and services in
the future. [text: E p. 31; MA p. 31; MI p. 31]

26. Describe the adjustments in the production possibilities curves in each of the following
situations for the U.S. economy.
(a) the economy moves from full employment into a deep recession
(b) the economy makes great strides in eliminating discrimination

157
Technology, R&D, and Efficiency

(c) the end of the cold war leads to cuts in military spending
(d) Congress significantly increases government spending for health and education
(a) The economy begins at a point on the curve but with recession there is
unemployment and the economy now operates at a point in the area inside the curve,
indicating that production is less than that which is possible because some resources are
not being used.
(b) Eliminating discrimination would move the economy from a point inside its
production possibilities curve toward a point on the curve.
(c) If the curve is illustrating the tradeoff between private spending and government
spending (or between military and consumer goods), then this should mean a movement
along the curve in the direction of more private or consumer production and less military
production. Government spending in general could decrease, but if that were not the
case, then the government might simply shift some funds from the military to other
types of government spending and the point would not necessarily move at all on a
curve depicting the tradeoff between government and private spending.
(d) Movement depends on where the money is coming from. If the money comes from
increased taxes or borrowing, then there is a movement along the curve away from
private spending and toward public spending. If the money comes from other
government programs and the curve is illustrating government versus private spending,
the amount of total government production would not necessarily change, so the point
could remain at the same spot on the curve. [text: E pp. 32-33; MA pp. 32-33; MI pp.
32-33]

28. Explain the term “laissez faire capitalism.”

Pure capitalism is sometimes called “laissez faire” capitalism which is a French term for
“let it be.” In pure capitalism the government’s role is limited to protecting private
property and establishing the legal framework for free enterprise and free markets to
function. [text: E p. 33; MA p. 33; MI p. 33]

B. Answers to Short-Answer, Essays, and Problems, chapter 3

New 4. Give three explanations for the law of demand:

First, it is explained by common sense. People tend to buy more of a product at a lower
price than at a higher price. Second, there is diminishing marginal utility: a decrease in
satisfaction that results with an increase in the amounts of a good or service. The
second unit of a good yields less satisfaction (or utility) than the first. Third, there are
income and substitution effects. With an income effect, a lower price increases the
purchasing power of money income, enabling you to buy more at lower price. With a
substitution effect a lower price gives an incentive to substitute the lower-priced good
for a now relatively high-priced good. [text: E p. 41; MA p. 41; MI p. 41]

7. List five basic determinants of market demand that could cause demand to decrease.
(The text mentions seven possibilities.)

(a) Consumers’ tastes become less favorable toward the item.


(b) The number of buyers decreases.
(c) Incomes fall and the item is a normal good.
(d) Incomes rise and the item is an inferior good.
(e) A decrease in the price of a substitute product.
(f) An increase in the price of a complementary product.
(g) Consumers expect lower prices in the future. [text: E pp. 43-46; MA pp. 43-46; MI
pp. 43-46]

158
Technology, R&D, and Efficiency

12. Suppose a producer sells 1,000 units of a product at $5 per unit one year, 2,000 units at
$8 the next year, and 3,000 units at $10 the third year. Is this evidence that the law of
demand is violated? Explain.

No. The law of demand shows the relationship between price and quantity demanded.
In general, as price falls the quantity demanded will increase. One of the assumptions,
however, is that all other things are equal or held constant. In this case, this assumption
may have been violated and that is why it seems there is a positive relationship between
price and quantity. The most likely explanation for the set of events is that demand for
the product increased from one year to the next. IF that was true, then price would rise
and the equilibrium quantity would increase. [text: E. pp. 41-42; MA pp. 41-42; MI pp.
41-42]

13. What effect should each of the following have upon the demand for portable CD-ROM
players? Explain your reasoning in each case.
(a) the development of improved, low-priced DVD players that compete with CD-ROM
players
(b) an increase in population and incomes
(c) a substantial increase in the number and quality of CD-ROM music
(d) consumer expectations of substantial price increases in CD-ROM players
(a) Would cause a decrease in demand for CD-ROM players, assuming that DVD
players are substitutes for CD-ROM players.
(b) Would cause an increase in demand because there are more consumers and they
have more income to spend. This assumes that CD-ROM players are a normal good and
more would be bought with higher incomes.
(c) Should increase demand since increased number and quality of CD-ROM music’s
variety of programs should make owning a CD-ROM player more desirable.
(d) Should increase current demand because expectations about the future have changed
and may prompt them to “buy now” to beat the future price increase. [text: E pp. 43-45;
MA pp. 43-45; MI pp: 43-45]

New 15. Describe and give a reason for the law of supply.

The law of supply indicates that producers will produce and sell more of their product at
a high price than at a low price. This means that there is a direct relationship between
price and quantity supplied. The basic explanation is that, given product costs, a higher
price means greater profits and thus more incentive for business to increase the quantity
supplied. [text: E p. 47; MA p. 47; MI p. 47]

17. The U.S. Congress is considering passing an excise tax that would increase the price of
a pack of cigarettes by $1.00. What would be the likely effect of this change on the
demand and supply of cigarettes? What is likely to happen to cigarette prices and the
quantity consumed if the tax bill is enacted?

In the short run, the excise tax would decrease the supply of cigarettes because in
essence it increases the cost of production. The decrease in supply would increase the
price of cigarettes and decrease the quantity of cigarettes consumed. The demand for
cigarettes would not change, but the quantity demanded would decrease. [text: E pp.
48-49; MA pp. 48-49; MI pp. 48-49]

19. Newspaper item: “Due to lower grain prices, consumers can expect retail prices of
choice beef to begin dropping slightly this spring with pork becoming cheaper after
midsummer,” the Agriculture Department predicted. “This reflects increasing supply,”
the department said. Does the statement use the term “supply” correctly? What effects
might this announcement have on consumer demand? Explain.

159
Technology, R&D, and Efficiency

The announcement does use the term “supply” correctly because the drop in price
predicted is a result of lower resource (grain) prices. This means that producers of beef
and pork will lower prices for each quantity on the existing supply schedule assuming
“all other things remain equal.”

Consumer demand at present might decrease as consumers wait to make big purchases
of beef and pork in the future when prices are predicted to drop. By spring, if beef
prices drop, there should be an increase in the quantity of beef demanded and probably a
decrease in the demand for pork, which is a substitute for beef. By midsummer, if pork
prices drop, there will be an increase in the quantity of pork demanded, and depending
on what is then happening with beef prices, a decline in the demand for beef. If beef
prices had continued to fall, it is hard to say whether there would be much of a change in
demand due to the price of the substitute pork falling. More likely, there would be only
a movement along the curve for beef if the price continued to fall. [text: E pp. 43-49;
MA pp. 43-49; MI pp. 43-49]

21. Given the products below and the events that affect them, indicate what happens to
demand, supply, equilibrium quantity, and equilibrium price. Identify the determinant
of demand and supply that causes the shifts.
(a) Calculators. More schools require students to buy and use calculators; improved
productivity shortens the time it takes to make calculators.
(b) Gasoline. Oil production declines due to a crisis in the Middle East; people take
more car vacations and drive more.
(c) New homes. The average incomes fall as the economy moves into recession; the
productivity of home construction workers and builders increases.
(a) The demand for calculators increases because of an increase in the number of
buyers. The supply of calculators increases because of a fall in resource prices
(productivity reduces resource costs). The equilibrium quantity increases, but what
happens to the equilibrium price is indeterminant and depends on the magnitudes of the
shifts.
(b) The supply of gasoline increases because of a rise in resource price (oil prices
increase due to a cutback in production). The demand for gasoline increases due to an
increase in the taste for taking driving vacations. The equilibrium price increases, but
what happens to the equilibrium quantity is indeterminant and depends on the
magnitudes of the shifts.
(c) The demand for new homes decreases because of a decline in consumer incomes.
The supply of new homes increases because of a fall in the price of labor resources
(productivity increases reduce resource costs). The equilibrium price decreases, but
what happens to the equilibrium quantity is indeterminant and depends on the
magnitudes of the shifts.

27. In the space below each of the following, indicate the effect [increase (+), decrease (–)]
on equilibrium price (P) and equilibrium quantity (Q) of each of these changes in
demand and/or supply.
P Q
(a) Increase in demand, supply constant ________ ________
(b) Increase in supply, demand constant ________ ________
(c) Decrease in demand, supply constant ________ ________
(d) Decrease in supply, demand constant ________ ________
(a) +, +; (b) –, +; (c) –, –; (d) +, – [text: E pp. 52-53; MA 52-53; E pp. 52-53]

B. Answers to Short-Answer, Essays, and Problems, chapter 4

New 2. Why is the right of private property an essential characteristic of a market system?

160
Technology, R&D, and Efficiency

Property rights are important because they give people private ownership of resources.
This ownership encourages them to make the best use of those resources because they
will benefit if they do. Property rights also encourage people to take the risk of
investing in capital goods or to develop new innovation or products because they will
receive the benefits from this activity if they are successful. Property rights offer
incentives for people to maintain their property to preserve its value and invest in it if
they think they can increase its value. Property rights also facilitate exchanges by
making clear who owns a resource so that there is no question about the legitimacy of
the transaction. [text: E pp. 59-61; MA pp. 59-61; MI pp. 59-61]

3. What role does freedom play in capitalism? How important is it to the operation of a
competitive market economy?

In capitalism, two essential freedoms are the freedom of enterprise and freedom of
choice. Business must be free to get economic resources, organize them to produce
products, and sell them in the market. Businesses must also be free to enter or leave an
industry. Freedom of choice gives business owners, resource owners, and consumers
freedom to act to advance their own self-interest. Business owners are free to buy and
sell property. Labor resources (workers) can enter or exit any line of work for which
they are qualified. Consumers are free to purchase goods and services as they choose
based on their budget constraints. Without these freedoms it would be impossible for a
market economy to exist. [text: E p 60; MA p. 60; MI p. 60]

New 4. Explain the importance of self-interest in the operation of a market system.

Self-interest is the motivating force behind both the demand and supply sides of the
product and resource markets in a capitalist system. Entrepreneurs engage in production
with the expectation of making a profit; workers seek the best wage and working
conditions they can get; consumers allocate their spending in order to maximize their
satisfaction. [text: E pp. 60-61; MA pp. 60-61; MI pp. 60-61]

New 5. What is the importance of competition in relation to self-interest in a market system?

Competition is important as the force which tempers or regulates the greed or self-
interest of the producers, sellers, workers, and buyers. If producers make too much
profit, competitors will arise to take advantage of the opportunity and as production
increases, prices and profits will fall. If workers demand too much, employers will seek
other workers who are willing to work for less. If buyers are not willing to pay a fair
price for the product, producers will sell the product to someone who is. In each case it
is competition or the threat of competition that tempers the greed of the economic
“player.” [text: E p. 61; MA p. 61; MI p. 61]

11. What are the economic advantages of specialization?

The advantages come from increased productivity or output per worker as specialized
workers gain skills in performing one task. Therefore, more is produced with the same
amount of resources as before specialization. [text: E p. 62; MA p. 62; MI p. 62]

16. Suppose Tom, Dick, and Harry live in a barter economy. Tom produces wine, Dick
bakes bread, and Harry makes cheese. Tom wants some bread to go with his wine and
is willing to trade 1 gallon of wine for two loaves of bread. Dick wants some cheese to
go with his bread and is willing to trade one loaf of bread for one-half pound of cheese.
Harry doesn’t want bread, but wants some wine to go with his cheese and is willing to
trade cheese for one gallon of wine. It is not possible for all three of them to meet
together at one time.
(a) Explain how this situation illustrates the difficulty with a barter economy.

161
Technology, R&D, and Efficiency

(b) Devise a money system using precious stones where four stones are equivalent in
value to one gallon of wine. In other words tell how much bread and cheese would be
worth in terms of stones in this economy. In this system, how much cheese must Harry
sell in order to buy one gallon of wine?
(a) The lack of coincidence of wants is illustrated in this situation. Tom wants the
bread that Dick has, Dick wants Harry’s cheese, and Harry wants Tom’s wine but for
any two of them there is no incentive to trade for something each does not want. In
other words, there is no coincidence of wants.
(b) If precious stones could be used as a medium of exchange, Tom could receive four
stones for each gallon of wine. According to their initial exchange values, he should be
willing to pay two stones for each loaf of bread. Dick would then have four stones to
purchase one pound of cheese and Harry could buy wine with the four stones he earned
from selling this cheese. In terms of stone prices, bread is worth two stones per loaf and
one gallon of wine and one pound of cheese are each worth four stones in this economy.
[text: E pp. 62-63; MA pp. 62-63; MI pp. 62-63]

18. Distinguish between normal and economic profits.

A normal profit can be explained as the normal return that an entrepreneur must receive
to cover the cost of the functions he or she performs in organizing and combining
resources in the firm. It is a cost payment for the entrepreneurial ability and time
invested in the business. An economic profit is any return above all costs including the
normal profit or return that the entrepreneur must receive. It is sometimes called “pure”
profit because it is over and above the return necessary to operate the business. [text: E
p. 64; MA p. 64; MI p. 64]

19. Assume that a firm can produce product A, product B, or product C with the resources it
currently employs. These resources cost the firm a total of $100 per week. Assume, for
the purposes of this problem, that the firm’s costs cannot be changed. The market prices
and the quantities of A, B, and C these resources can produce are given as follows.

Product Market price Output Profit


A $14.00 10 $_____
B 9.00 11 _____
C .50 300 _____

(a) Compute the firm’s profit when it produces A, B, or C and enter these data in the
table.
(b) Which product will the firm produce?
(c) If the price of A rose to $16, which product will the firm produce?
(d) If the firm produces A at a price of $16, what would tend to happen to the number
of firms producing product A?

162
Technology, R&D, and Efficiency

Product Market price Output Profit


A $14.00 10 $40
B 9.00 11 –1
C .50 300 50

(a) Profit for A will be $40; for B will be –$1 (loss); for C will be $50.
(b) Firm will produce product C.
(c) If the price of A rises (assuming the firm can sell all of its output at the new price), it
would make a profit of $60 on A and so would produce product A.
(d) If other firms face similar costs, more firms would enter industry A to take
advantage of the higher economic profits. [text: E pp. 64-65; MA pp. 64-65; MI pp. 64-
65]

23. Why does competition force firms to use the least-cost, most efficient, productive
techniques?

If firms do not use the most efficient techniques, their rivals who do use more efficient
methods will be able to charge lower prices and potentially take buyers from the less
efficient seller. If the less efficient firm has to lower prices to match the competition
without improving efficiency, it will eventually realize losses and in the long term will
have to leave the industry unless it, too, uses the most efficient production methods.
[text: E p. 66; MA p. 66; MI p. 66]

26. What is meant by the guiding function of prices?

The guiding function of prices refers to the response of producers to changes in prices.
With a given supply schedule, if demand increases, the market equilibrium price will
rise and producers will respond to that by producing an increased quantity of the
product. Also, there may be entry of firms into the industry. Conversely, if demand
decreases, the market equilibrium price will fall and producers will respond to that
change by producing a decreased quantity of the product. There may also be an exodus
of firms from the industry. The market system is a communications system and prices
are the way information is communicated and incentives are given to take action. [text:
E p. 67; MA p. 67; MI p. 67]

B. Answers to Short-Answer, Essays, and Problems, chapter 5

1. Name the categories used in describing the functional distribution of income. Tell
which type receives the largest share and approximately how much this is in fraction or
percentage terms.

The categories are wages and salaries, proprietors’ income, corporate profits, interest,
and rents. Wages and salaries are by far the largest share comprising seventy percent of
the total (70%). (If we recognize that part of proprietor’s income is also a form of
income to labor resources, then labor’s share is even larger, perhaps about 80%.) [text:
E pp.73-74; MA pp.73-74; MI pp.73-74]

3. How do households dispose of their income? Identify the three major ways income is
disposed of and the relative share for each category.

Household income is disposed of in three ways. First, income is paid to the government
as taxes. Second, some household income is saved. The great majority of household
income, however, is used for personal consumption of goods and services. In 2000,
84% of income went for personal consumption expenditures, 16 % went for personal
taxes and 0% went to personal saving. [text: E pp. 74-75; MA pp. 74-75; MI pp. 74-75]

163
Technology, R&D, and Efficiency

5. Describe the business population. What is the difference between a plant, a business
firm, and an industry?

The business population of the American economy consists of three major types of
entities. A plant is a physical structure that produces a product. A business firm is an
organization which owns and operates plants. (Multiplant firms may be horizontally
integrated, vertically integrated, or conglomerates.) An industry is a group of firms
producing the same or similar goods or services. [text: E p. 76; MA p. 76; MI p. 76]

8. What are the three principal legal forms of business firms? What are the advantages and
disadvantages of each?

The three principal legal forms of business firms are the proprietorship, partnership and
corporation. Each has special characteristics and advantages and disadvantages. The
proprietorship is easy to form, lets the owner be boss, and allows great freedom. The
disadvantages are the lack of access to large amounts of financial capital, the difficulty
of managerial specialization, and the unlimited liability of the owner. The partnership is
also easy to form, and allows for more access to financial capital and permits more
managerial specialization. The potential disadvantages are that partners may disagree,
there are still limits to financial capital or managerial specialization, continuity of the
firm over time is a problem, and there is unlimited liability for partners. The
corporation can raise financial capital through the sale of stocks and bonds, has limited
liability for owners, can become large in size, and has an independent life. The chief
disadvantages are the double taxation of some corporate income, potential for abuse of
this legal entity, and legal or regulatory expenses. There can also be a principal-agent
problem with the separation of ownership and control of the firm. [text: E pp. 76-78;
MA pp 76-78; MI pp. 76-78]

11. Define what is meant by a corporation and give two reasons why corporations dominate
the American business sector.

Corporations are legal business organizations which are distinct and separate entities
from the individuals who own them. As such, corporations are treated as legal persons
that can acquire resources, own assets, produce and sell products, incur debts, extend
credit, sue and be sued, and carry on all functions which any other form of enterprise
performs.

Corporations dominate the business sector in terms of sales because they are the most
effective form of business for raising money capital with their ability to issue stocks and
bonds. Therefore, the largest businesses tend to be corporations. Stockholders also
have the advantage of limited liability which is important when there is much at risk, so
again the larger firms have an incentive to incorporate. [text: E pp. 77-78; MA pp. 77-
78; MI pp. 77-78]

13. Explain what “separation of ownership and control” of the modern corporation means.
How is this situation a principal-agent problem?

In many large modern corporations there are thousands of stockholders, each with a very
small share of the company. They have little or no power to make decisions in the
corporation and only have the power to cast their share votes on a few decisions at the
annual meeting of shareholders. The day-to-day decisions involving the corporation are
made by the paid professional managers. In many cases, even the board of directors
elected to represent the shareholders are largely at the mercy of management
recommendations when it comes to making policy regarding the company. Thus, the
shareholders of a large corporation are separated from the control of the corporation
they own. This situation is an example of a principal agent problem where the interests

164
Technology, R&D, and Efficiency

of the principals (stock holders) differ from the interests of agents (managers). [text: E
p. 78; MA p. 78; MI p. 78]

14. Classify each of the following specific policy actions in terms of the five basic functions
of government. Justify your classification in each case.
(a) Government expenditures for military and space research
(b) Amending the existing social security system to provide “catastrophic health
insurance” for long illnesses
(c) The establishment of the Federal Trade Commission to police false and misleading
advertising
(d) State subsidies to local school districts
(e) A Federal excise tax on gambling
(f) A program to distribute surplus agricultural products to poor families
(a) This is a reallocation of resources from the private sector to the public sector to
provide for public goods in the form of military and space research which is both
indivisible and would have a free-rider problem if left to private sector production.
[text: E pp. 80-82; MA pp. 80-82; MI pp. 80-82]
(b) This is an example of redistribution of income from all those who are able to pay a
portion of the amount needed for this insurance fund to those who would be unable to
pay for a long catastrophic illness. One might also argue that there is some degree of
reallocation of resources from private spending to public health insurance because of the
perceived spillover benefits in providing health care to those who cannot afford it as
well as those who can. There are spillover benefits to promoting health and welfare
among all families, not just those who are able and willing to pay the cost of private
catastrophic health insurance. [text: E pp. 79-80; MA pp. 79-80; MI pp. 79-80]
(c) This is primarily an example of government’s role in providing the legal framework
conducive to a healthy market system. It is also part of the government’s role in
maintaining competition. Without such regulation, some firms may gain monopoly
control by unfair trade practices. [text: E p. 79; MA p. 79; MI p. 79]
(d) This is an example of reallocating resources to subsidize the cost of public
education because there are substantial spillover benefits for the entire state when there
is a good educational system in place. It also is an example of the government’s role in
redistributing income if the subsidies to poor school districts exceed tax collections from
those districts. [text: E pp. 80-82; MA pp. 80-82; MI pp. 80-82]

(e) This is an example of the government’s role in deterring an undesirable activity. It


also is an example of redistribution of income from those who earn income from
gambling to the public sector. There is some feeling that income from gambling is a
type of unearned income and therefore, perhaps less deserved than other forms of
income. Therefore, from a fairness perspective redistribution from those private
individuals who earn income from gambling to public programs is viewed as desirable.
[text: E pp. 80-82; MA pp. 80-82; MI pp. 80-82]
(f) This program redistributes income to two groups from taxpayers. By buying the
surplus agricultural products in the first place, the government is redistributing income
from taxpayers to farmers. Then distribution of this surplus to poor families is an in-
kind redistribution of income from taxpayers to poor families. There is some
reallocation of resources to the agricultural sector from other private sector production
since government purchase of surplus products increases demand in agriculture. [text E
pp. 79-80; MA pp. 79-80; MI pp. 79-80]

15. “A society has every right to reject the distribution of income which is automatically
formed by the market system.” Do you agree? Justify your position.

One would probably agree with this statement for the reasons given under the section in
the chapter on the government’s role in redistributing income. A democratic society has

165
Technology, R&D, and Efficiency

a sense of fairness and equal opportunity for all. Therefore, we feel we should provide
for the less fortunate among us.

Also, one could justify this position based on more selfish reasons. It is in society’s
self-interest to have a healthy, educated productive populace. The market system will
not always provide for this on its own because of the possibility of spillover costs such
as pollution or spillover benefits such as education. [text: E pp. 79-82; MA pp. 79-82;
MI pp. 79-82]

17. Explain what is meant by the term “market failure” by giving two major types of such
failures. Then explain the reasons behind each type of failure.

A market failure is a situation where the competitive market system on its own would
either (1) produce the “wrong” amount of certain goods and services as is the case
where there are spillover costs or benefits, or (2) fail to allocate any resources to the
production of certain goods and services referred to as “public” or “social” goods.

Where spillover or external costs exist, the market system without any intervention will
allow some of the costs of private production or consumption to “spill over” onto third
parties. There is no market incentive to absorb all of the costs of production if there is a
way to get someone else to pay for them. In such cases of spillover costs, the market
cost and price of the product will be too low and thus the quantity produced too high in
terms of its true cost of production. Resources are overallocated to the production of
such goods. Likewise, in the case of spillover benefits there is no incentive for the
private consumer to pay for any more than the benefits that they receive as individuals.
However, there may also be benefits that exist for society as a whole. Relying on the
market alone will not provide the additional benefits that society would like from these
products, and resources are underallocated where spillover benefits exist.

In the case of public goods and services, it is impossible to divide the product into
individual saleable units for consumers to purchase and, additionally, the exclusion
principle doesn’t apply. That is, even if the product could be sold, consumers would not
want to purchase it because “free riders” would take advantage of their purchase.
People could receive the benefits without paying for the product. In such cases, there is
no incentive for individuals to purchase the item on their own even though they may
desire the good or service. These are public goods and services and their provision is
made possible through government purchases where tax revenues can be used to provide
the product. [text: E pp. 80-82; MA pp. 80-82; MI pp. 80-82]

18. Define spillover cost and give an example.

When production or consumption of a commodity inflicts costs on third parties without


compensation there is a spillover cost. Examples of spillover costs of production are
primarily various forms of pollution. Spillover costs of consumption might include such
things as a noisy neighbor whose habits lower neighborhood property values. [text: E
pp. 80-81; MA pp. 80-81; MI pp. 80-81]

20. Explain the exclusion principle and the free-rider problem and how they are related.

Most private goods and services are subject to the exclusion principle which is the idea
that those who pay for the product are the ones who get it, but those who don’t pay for it
are excluded from the benefits provided by that product.

The free-rider problem exists when the exclusion principle does not apply. That is,
people can receive benefits from a product without contributing to its costs. For

166
Technology, R&D, and Efficiency

example, citizens receive benefits from a country’s national defense whether or not they
pay for it. [text: E pp. 81-82; MA pp. 81-82; MI pp. 81-82]

24. Differentiate between government purchases of goods and services and government
transfer payments.

Transfer payments are government payments such as unemployment compensation,


social security benefits, welfare payments, and Medicare benefits. They are called
transfer payments because they are essentially transferring funds from revenue accounts
to those entitled to receive them. No current production is being performed by the
recipients. Government purchases of goods and services such as military products and
public employees’ salaries represent payment for the goods and services produced for
the public sector. [text: E pp. 84-85; MA pp. 84-85; MI pp. 84-85]

25. How is total government spending defined? What is its size in the economy?

Total government spending consists of government purchases of goods and services and
transfer payments. Each has a different effect on the economy. Purchases are
exhaustive because they directly use the economy’s resources while transfers are
nonexhaustive. Total government spending is equal to about one-third of domestic
output. [text: E pp 84-85; MA pp. 84-85; MI pp. 84-85]

New 26. List the four main categories of Federal spending.

The four main categories of Federal spending are pensions and income security
programs, national defense, health care and interest on the Federal debt. In 1999, about
39% of Federal expenditures went for pensions and income security, 19% went for
health care, 16% went for national defense, and 14% went for interest on the national
debt. [text: E p. 85; MA p. 85; MI p. 85]
New 27. List the four major sources of Federal revenues.

The four main sources of revenue are the personal income tax, payroll taxes, corporate
income taxes, and excise taxes. In 1999, the personal income tax accounted for 48% of
tax revenues. Payroll taxes accounted for 35% of tax revenues. Corporate income taxes
were 10% of revenues. Excise taxes were 4% of tax revenues. [text: E pp. 86-87; MA
p. 86-87; MI pp. 86-87]

28. Explain the difference between marginal and average tax rates.

Average tax rates are calculated by taking the total tax paid and dividing by the total
income. In other words, the comparison is between two dollar amounts: total tax paid
and total income. On the other hand, the marginal tax rate is a comparison of the
difference in tax paid relative to the change in income which causes that differential. In
other words, it is a comparison of two incremental amounts or differences. Marginal tax
rates are calculated by taking the change in the tax paid and dividing by the associated
change in income. [text: E pp. 86-87; MA pp. 86-87; MI pp. 86-87]

B. Answers to Short-Answer, Essays, and Problems, chapter 6

2. “The best indicator of the importance of a nation’s world trade is the ratio of its exports
plus imports to the size of its domestic output.” Critically evaluate.

Certainly this is a good indicator, but not the only one. If the ratio is very large, that
would indicate that world trade is very important in that nation. However, in a nation
such as the United States, when imports and exports are summed and compared to GDP,
the ratio is less than 30%. Yet trade is still very important since we depend almost

167
Technology, R&D, and Efficiency

entirely on imports for some raw materials which are not available in the United States.
[text: E pp. 94-95; MA pp. 94-95; MI pp. 94-95]

3. Suppose all American international trade suddenly ceased. What would be the
consequences domestically? Internationally?

Our GDP would be affected to a great extent. In 2000, exports were 12% of GDP —
too great a proportion to be absorbed quickly in the domestic market. Imports were
about 17% of GDP. The loss of imports would mean a decline in living standards and
increased purchase of higher priced U.S. goods where they could be produced. It would
also result in a loss of some products altogether such as coffee, tea, nickel, tin, and
others. The decline in GDP would mean a decline in productivity and employment in
those industries affected. These conditions would probably cause further declines in
output and employment in other industries as well.

Internationally, the world would also see a decline in living standards since the United
States is the world’s largest importer and exporter. Without U.S. expenditures on their
products, other nations would suffer. Without U.S. exports, their living standards would
decline. [text: E pp. 94-96; MA pp. 94-96; MI pp. 94-96]

9. Explain the principle of comparative advantage in nontechnical terms.

If two nations do not have identical costs of production, then each nation should
specialize in production of the good or goods which it can produce at the relatively
lower cost. The key term is “relative” since one nation may be able to produce
everything at an absolutely lower cost.

It is sometimes easier to explain in terms of individuals. If a doctor is a very good house


painter, and can paint her house twice as fast as any professional painter, this does not
mean she should paint her own house from the standpoint of comparative advantage. A
doctor’s time is worth substantially more than a painter’s time, so from a relative
standpoint, it makes sense to hire the painter to paint the house even if it takes him twice
as long as the doctor. The doctor stands to gain economically by practicing medicine
during the time it would take her to paint the house. [text: E pp. 98-99; MA pp. 98-99;
MI pp. 98-99]

12. The countries of Macrostan and Micrastan have the production possibilities tables for
sheep and hogs shown in the tables below. Without trade Macro would produce at
Alternative D and Micra would also produce Alternative D. Note that the costs of
producing sheep and hogs are constant in both countries.

Macrostan’s Production Possibilities Table


Product Alternative
(lbs) A B C D E F
Sheep 25 20 15 10  5  0
Hogs  0  5 10 15 20 25

Micrastan’s Production Possibilities Table


Product Alternative
(lbs) A B C D E F
Sheep 20 16 12 8  4  0
Hogs  0  3  6 9 12 15

(a) In Macro, the cost of producing:


5 units of sheep is ______ hog units.
1 unit of sheep is ______ hog units.

168
Technology, R&D, and Efficiency

(b) In Micra, the cost of producing:


4 units of sheep is ______ hog units.
1 unit of sheep is ______ hog units.
(c) Which country has the comparative advantage in sheep production and which
country has the comparative advantage in hog production?
(d) If each nation specializes in the product where it has a comparative advantage and
trades with the other, what will be the limits to the terms of trade for each sheep unit?
(e) If the nations do not specialize and trade but remain at alternative D in Macrostan
and D in Micrastan, the combined production of Macro and Micrastan will be how many
sheep and how many hogs?
(f) However, if the two nations specialize, the combined production of Macro and
Micrastan will be how many sheep and how many hogs?
(g) What will be the total gain of sheep and hogs if the countries specialize and trade?

(a) In Macro, the cost of producing:


5 units of sheep is 5 hog units.
1 unit of sheep is 1 hog units.
(b) In Micra, the cost of producing:
4 units of sheep is 3 hog units.
1 unit of sheep is 3/4 hog units.
(c) Based on the opportunity cost of producing in each country, Macrostan has a
comparative advantage in sheep while Micrastan has a comparative advantage in hogs.
(d) If each nation specializes in production where it has a comparative advantage and
trades with the other, the limits to the terms of trade will be between 3/4 and 1 hog unit
for each sheep unit (or conversely between 4/3 and 1 unit of sheep for each unit of
hogs).
(e) If the nations do not specialize and trade but remain at alternative D in Macrostan
and D in Micrastan, the combined production of Macrostan and Micrastan will be 18
sheep and 24 hog units.
(f) However, if the two nations specialize, the combined production of Macro and
Micrastan will be 20 sheep and 25 hog units.
(g) Total gains from specialization will be 2 sheep and 1 hog unit (20-18 sheep and 25-
24 hogs). [text: E pp. 99-101; MA pp. 99-101; MI pp. 99-101]

14. What are the economic effects of a depreciation of the dollar on foreign exchange
markets?

Depreciation means that it takes more dollars to buy foreign currencies. This means that
foreign goods become more expensive to Americans and imports should decline as
Americans shift to spending more on U.S. products. It also means that foreign
consumers can obtain more dollars with their currency and therefore, foreign purchases
of American exports abroad should rise. Both of these outcomes should stimulate the
demand for American goods and expand the U.S. economic output. [text: E pp. 102-
103; MA pp. 102-103; MI pp. 102-103]

23. Why do governments often intervene in international trade to restrict imports and
expand exports?

Governments may intervene in trade between nations because they mistakenly think of
exports as helpful and imports as harmful for a national economy. In fact, there are
important gains from trade in the form of the extra output obtained from abroad. Trade
makes it possible to obtain a product at a lower cost than would be the case if they were
produced using domestic resources, and the earnings from exports help a nation pay for
these lower-cost, imported products. Another reason why governments interfere with
free trade is based on political considerations. Groups and industries seek protection
from foreign competition through tariffs and import quotas, or other kinds of trade

169
Technology, R&D, and Efficiency

restrictions. The costs of trade protectionism are hidden from consumers of the
protected product so there is little opposition to demands for protectionism. [text: E pp.
104-105; MA pp. 104-105; MI pp. 104-105]

28. What is NAFTA? What have critics and defenders said about it?

NAFTA is the North American Free Trade Agreement. It formed a trade bloc among
the United States, Canada, and Mexico in 1993. This agreement eliminated tariffs
among these three nations. Critics in the United States say that it has decreased jobs and
that it may become a base for tariff-free imports from Asian nations. Defenders say that
it will allow each nation to specialize according to comparative advantage. The act will
encourage more investment in Mexico, thus increasing Mexico’s productivity and
national output. This result will give Mexico more funds to purchase goods and services
from the United States, thus increasing jobs in this nation. [text: E p. 108; MA p. 108;
MI p. 108]

CHAPTER 7 Measuring Domestic Output, National Income, and the Price Level

3. What is the definition of GDP? How would the value of output produced at an
American-owned factory in the U.S. and a foreign-owned factory in the U.S. be treated
in GDP accounting?

GDP is a measure of the total market value of all final goods and services produced
within a country in a year period. The value of output produced at an American-owned
factory in the U.S. and a foreign-owned factory in the U.S. would both be treated as part
of domestic output in GDP accounting. Thus, GDP represents all domestic production.
[text: E pp. 117-118; MA pp. 117-118]

4. Why is GDP a monetary measure?

GDP is a monetary measure to make it possible to compare the relative worth of a


diverse collection of goods and services over time. It is not possible to count the
number of goods and compare them because the types of goods change over time. It is
possible to count the number of goods and attach monetary values to them to reflect
their relative worth and then compare the value of the output at different points in time.
[text: E p. 117; MA p. 117]

5. Explain the difference between final and intermediate goods, and give an example of
each.

Goods are considered to be “final goods” when they reach their final point of sale in the
given year. The textbook example of a final good is the wool suit bought from a retail
store by one of its customers. “Intermediate goods” are goods produced during the year
that will be used in the production of something else. In the wool suit example, the
value of the wool sold by the sheep rancher was not counted when it was sold to the
cloth manufacturer because it would be used in the process of producing the cloth and
later the suit. In other words, at the wool stage, the product had not reached its final
point of sale for that year.

Other examples would include all sorts of raw materials and wholesale goods as
intermediate goods. Any consumer purchase of a good or service domestically
produced in the given year would be a final expenditure in the GDP accounts. [text: E
pp. 117-118; MA pp. 117-118; MI]

170
Technology, R&D, and Efficiency

6. Why do economists worry about “multiple counting” and calculate only the “value
added” in the production process?

Only the value of final goods is included in the calculation of GDP because that value
includes the value of the production of all intermediate goods. To include the value of
final goods and all intermediate goods would lead to a multiple counting or
overstatement of the size of GDP.

To make the GDP calculation, economists only count the “value added” at each stage of
the production process. Value added is the market value of a firm’s output minus the
value of inputs bought from other firms. Summing the value of the contribution made
by each firm at each stage of the production process enables economists to determine
GDP. [text: E pp. 117-118; MA pp. 117-118]

7. What is the value added by all the firms A–E from the production of a product as
described below? What did each firm add separately in value and what does it total?

Stage of production Sales value of product


Firm A $1,600
Firm B 2,500
Firm C 3,700
Firm D 5,200
Firm E 7,600

The value added by all firms is $7,600, or the final sales value. Firm A: added $1,600.
Firm B: added $900. Firm C: added $1,200. Firm D: added: $1,500. Firm E: added
$2,400. The value added by all firms totals $7,600 and equals the final sales value by
Firm E ($7,600). [text: E p. 117-118; MA p. 117-118]

8. Identify at least four transactions and other variables which are not included in the Gross
Domestic Product.

(1) Nonmarket transactions such as do-it-yourself projects and homemaker services are
excluded; (2) purely financial transactions such as the purchase and sale of stocks and
bonds are not included; (3) secondhand sales are not included; (4) public and private
transfer payments are excluded; (5) intermediate goods are not included until their point
of final sale that year. [text: E pp. 117-118; MA pp. 117-118]

9. Define the four categories of expenditures which comprise GDP.

(1) Personal consumption expenditures including consumer spending on durable goods,


nondurable goods, and services; (2) gross private domestic investment which includes
business purchases of machinery, equipment and tools, business and residential
construction, changes in business inventories; (3) government purchases of goods and
services, excluding government transfer payments; (4) the value of net exports which is
the value of exports of goods and services minus the imports of goods and services.
[text: E pp. 119-122; MA pp. 119-122]

10. Give the three categories which comprise gross investment and explain the difference
between them.

Gross investment includes (1) final purchases of machinery, equipment, and tools by
businesses; (2) all construction including residential; (3) changes in business
inventories. The first group restates the definition of investment goods; the second item
is houses and buildings that can yield an income return; the third group is “unconsumed
output” and is therefore part of investment. [text: E pp. 119-120; MA pp. 119-120]

171
Technology, R&D, and Efficiency

11. Net investment can be positive, negative, or zero, but gross investment can never be less
than zero. Explain.

This is true by definition. Gross investment refers to the amount of investment spending
on capital goods before allowance is made for depreciation. If any capital spending
takes place, then the amount has to be positive. If none takes place, gross investment
could be zero, but it could never be less than zero. [text: E pp. 120-121; MA pp. 120-
121]

12. Explain what is and what is not included in government purchases in GDP.

Government purchases are government expenditures for goods and services the
government consumes in producing public goods. These expenditures are for final
goods and all direct purchases of resources such as labor. It also includes expenditures
for social capital such as highways and buildings that have a long life. What is not
included are government expenditures for transfer payments such as social security or
welfare because these expenditures generate no production. [text: E pp. 120-121; MA
pp. 120-121]

13. Define net exports.

The value of net exports equals the value of exports minus the value of imports of goods
and services. If imports exceed exports, this will be a negative value. [text: E p. 121;
MA p. 121]

19. Which of the following are included and which are excluded in calculating this year’s
GDP? Explain in each instance.
(a) A monthly scholarship check received by an economics student
(b) The purchase of a new corncrib by a farmer
(c) The purchase of a used tractor by a farmer
(d) The cashing in of a savings bond
(e) The services of a mechanic in fixing the radiator in his own car
(f) Social security checks received by a retired person
(g) An increase in business inventories
(h) Government purchase of missiles
(i) A barber’s income
(j) Income received from interest on a corporate bond
(k) Cash received from selling a corporate bond

(a) Scholarships are viewed as awards for past performance and would not be included
in current production. They don’t represent income earned by providing a productive
resource as defined in the GDP accounts. [text: E p. 118; MA p. 118]
(b) Is included because it represents investment. It is a final good that was produced in
the current year. [text: E pp. 119-120; MA pp. 119-120]
(c) Not included because it was counted when it was new. [text: E p. 118; MA p. 118]
(d) Not included because it represents a financial transaction only. [text: E pp. 118;
MA p. 117]
(e) Not included because it is not a market transaction. [text: E pp. 117, 130; MA pp.
117, 130]
(f) Not included because it is a transfer payment, not payment for current productive
services. [text: E p. 118; MA p. 118]
(g) Is included as part of business investment. [text: E pp. 119-120; MA pp. 119-120]
(h) Is included as part of government spending on goods. [text: E pp. 120-121; MA pp.
120-121]
(i) Is included because it is payment for productive services (barbering). [text: E p.
119; MA p. 119]

172
Technology, R&D, and Efficiency

(j) Is included because it is payment for use of capital resources during that year. [text:
E p. 122; MA p. 122]
(k) Not included because it represents a financial transaction only. [text: E p. 118; MA
p. 118]

20. How is a price index computed?

A price index is computed by comparing the total price of a “market basket” of goods
and services representing GDP in the given year to the total price of the same market
basket in the base year. The given year’s total price is divided by the base year’s total
price to arrive at the index figure. The index is expressed as a percentage, so if the base
year’s prices are higher than the given year, the index will be less than 100. If the given
year’s prices are higher than the base year, the index will be more than 100. The price
index for the base year will always be 100 since the numerator and denominator will be
the same when the given year and base year are the same. [text: E pp. 127-128; MA pp.
127-128]

21. Differentiate between nominal and real GDP.

Nominal GDP is the actual measured GDP in terms of current year dollars or prices
existing at the time the output was produced. Real GDP reflects the value of GDP after
it has been corrected for price changes compared to the price level in a reference year
(called the base year). A GDP price index is calculated each year to measure the level
of prices relative to the level of prices in the base year. This price index is then
expressed as a percentage of the base year price level. [text: E pp. 127-129; MA pp.
127-129]

23. The next four questions refer to the following price and output data over a five-year
period for an economy that produces only one good. Assume that year 2 is the base
year.
Units of Price
Year output per unit
1 16 $2
2 20 3
3 30 4
4 36 5
5 40 6

(a) If year 2 is the base year, give the price index for year 3.
(b) Give the nominal GDP for year 4.
(c) What is the real GDP for year 4?
(d) Tell which years you would deflate nominal GDP and which years you would
inflate nominal GDP in finding real GDP.

(a) 4/3 = 1.33 for a price index of 133


(b) Nominal GDP = 36 × $5 = $180
(c) Real GDP = 180/1.67 = $107.78 (approx.)
(d) You would deflate nominal GDP for years where the price index is more than 100,
that is, for years 3, 4, and 5. You would inflate the nominal GDP for year 1 since the
price index is less than 100 relative to the base year 2. [text: E pp. 127-129; MA pp.
127-129]

28. What is the consumer price index? How does it differ from the GDP index?

173
Technology, R&D, and Efficiency

The consumer price index (CPI) is a historical fixed-weight price index that measures
the ratio of the current price of a fixed, base-period market basket to the base period
price of the same market basket, multiplied by 100. It is designed to measure the cost of
a constant standard of living. The market basket includes some 300 goods bought by the
typical urban consumer.

The GDP price index compares the price (or cost) of goods and services that make up
GDP in a specific year to the price of the same set of goods in a reference year. The
GDP is a much broader measure that includes consumer goods and services, capital
goods, goods and services purchased by government, and goods and services entering
world trade. With the GDP index the weights or relative purchases of goods and
services are adjusted continuously and are not fixed as they are for the CPI. [text E pp.
128-130; MA pp. 128-130]

29. Discuss the merits and demerits of GDP as a measure of the economy’s output
performance and as a measure of its standard of living.

GDP is a reasonably accurate measure of output performance especially for purposes of


comparison with past performance and with other nations. As long as measurement is
done in a consistent manner from year to year and from country to country then such
comparisons are valid and useful.

As a measure of standard of living, GDP has more shortcomings, but there still is a
strong positive correlation between real GDP per capita and standard of living. As a
tool to compare living standards across countries GDP per capita is useful.

The shortcomings of GDP as a measure of standard of living include some cases where
GDP may overstate real output and some cases where GDP may understate it. In the
category of overstating the standard of living, GDP measures do not deduct for
environmental pollution, for product quality deterioration, and for production of what
one might call necessary evils like devices and services to protect against crime. In the
category of understating the standard of living, GDP measures do not reflect
improvements in product quality, increased leisure time, nonmarket production services,
and all of the production both legal and illegal which falls into the category of the
underground economy. [text: E pp. 130-131; MA pp. 130-131]

CHAPTER 8 Introduction to Economic Growth and Instability

2. Suppose an economy's real GDP is $50,000 in year 1 and $55,000 in year 2. What is the
growth rate of its GDP? Assume that population was 100 in year 1 and 105 in year 2.
What is the growth rate in GDP per capita?

$5,000/$50,000 or 10% in year 1. The per capita growth can be calculated as follows:
$500 per capita in year 1 ($50,000/100); $523.81 per capita in year 2 ($55,000/105). The
change is 23.81 compared to base of $500 or 23.81/500 = 4.77%. [text: E pp. 136-137;
MA pp. 136-137]

4. Explain why even small changes in the rate of economic growth are significant. Use the
"rule of 70" to demonstrate the point.

Small changes in the rate of growth can be very meaningful, especially for a country
where a fraction of a percent change in the growth rate may mean the difference between
starvation and hunger.

174
Technology, R&D, and Efficiency

Over a period of time small changes are cumulative in the same way that compound
interest payments are cumulative on a bank account. Using the rule of 70 to estimate the
time it takes to double GDP, we can see that a country whose growth rate is 5% takes 14
years to double its GDP, but a country whose growth rate is 3% may take nearly 10 years
longer to double its GDP or about 23.3 years. If these countries continued to grow at their
respective 5% and 3% rates, in 28 years the first country's GDP would be quadrupled,
whereas in the second country, it would take nearly 47 years to quadruple its GDP from
the current year. [text: E p. 137; MA p. 137]

5. Which is more important — increasing inputs of resources or increasing the productivity


of those inputs — as the main source of economic growth in the United States? Explain.

Both resource quantity and resource productivity contribute significantly to increased


economic growth in the United States. About a third of the increase in economic growth
comes from increases in resource input and about two-thirds from increases in resource
productivity. Clearly, it is not the quantity of resources that is most important but the
quality of resources and how they are used for production. [text: E p. 137; MA p. 137]

7. What is meant by the term business cycle? List the four phases of the business cycle.

The term business cycle refers to the recurrent ups and downs in the level of economic
activity extending over several years. The four phases of the business cycle are (1) the
peak phase at which business activity reaches a temporary maximum; (2) recession
which is a period of decline in total output, income, employment, and trade lasting six
months or longer; (3) the trough which is the bottom of the recession where output and
employment reach their lowest level for the period; and (4) the recovery phase is the
expansionary phase as output and employment rise toward full employment. [text: E pp.
138-139; MA pp. 138-139]

8. What phase of the business cycle is our economy experiencing at the present time?
Justify your answer.

This question might best be given as a group assignment. Direct your students to the
current news media. In particular, The Wall Street Journal has frequent articles on the
state of the economy as do news magazines such as Business Week. Even without
reading any articles, students could surmise the phase by finding out the current inflation
and unemployment rates and/or finding information on the latest report of “leading
indicators,” and comparing this information to similar statistics from the recent past.
[text: E pp. 138-139; MA pp. 138-139]

10. Which types of industries are hit hardest by a recession? Explain.

Industries specializing in consumer durable goods production and capital goods


production are hardest hit. When economic activity slows, people postpone purchases
of durable goods which worsens the downturn. Likewise, businesses postpone the
purchase of new equipment and plant expansion if they expect a downturn. These
slowdowns in business spending cause further deterioration in economic activity. [text:
E p. 140; MA p. 140]

13. If the population is 267 million, the labor force is 136 million, and the number measured
as unemployed is 6.8 million, what is the rate of unemployment?

Divide 6.8 million by 136 million to get 0.05; Multiply 0.05 by 100 to get 5.0% as the
rate of unemployment. [text: E pp. 140-141; MA pp. 140-141]

175
Technology, R&D, and Efficiency

15. What are two criticisms of the unemployment rate? How do these criticisms relate to
the overstating or understating of the unemployment rate?

First, the official data understates the extent of unemployment because it considers all
part-time employees as fully employed. Second, a worker must be actively seeking
work to be counted as unemployed, which also understates the level of unemployment.
Workers who are too discouraged to look for work are not considered part of the labor
force. [text: E p. 141; MA p. 141]

16. Explain the differences among the frictional, structural, and cyclical forms of
unemployment.

Frictional unemployment is the unavoidable and even positive type of unemployment


characterized by people who are searching for new jobs or waiting to take jobs in the
near future. Structural unemployment is unemployment due to changes occurring over
time in the structure of consumer demand or in technology which result in some skills
being in less demand or even becoming obsolete. It may also be due to a change in the
geographic distribution of job opportunities. Cyclical unemployment is the
unemployment caused by the recession phase of the business cycle. Economists place
special emphasis on this type of unemployment because they believe that economic
policy can reduce cyclical fluctuations in the economy and therefore can reduce cyclical
unemployment. [text: E pp. 141-142; MA pp. 141-142]

17. Define the “full-employment” or “natural” rate of unemployment and give its
approximate percentage rate as economists currently define it.

“Full employment” does not mean zero percent unemployment, but refers to that part of
unemployment viewed as being unavoidable. This is generally the sum of frictional and
structural unemployment and today is generally believed to be about 5.5%. [text: E pp.
142-143; MA pp. 142-143]

19. What is Okun’s law? Give an example of how it works.

Okun’s law refers to the relationship between the unemployment rate and the GDP gap
which was quantified in studies done by the economist Arthur Okun. His studies
indicated that for every 1 % that the actual unemployment rate exceeds the natural
unemployment rate, there is a 2% GDP gap. If the actual unemployment rate was 7.5%
and the natural rate was 5.5%, then there would be a 4% GDP gap. Thus, the ratio is 1:2
in terms of unemployment rate–GDP gap link. [text: E p. 143; MA p. 143]

21. Calculate the rate of inflation between Year 1 and Year 2. The price index in Year 1
was 124.0. It was 130.7 in Year 2.

Subtract 130.7 minus 124.0 = 6.7; Divide 6.7 by 124.0 (since that is the index in the
comparison year in this problem). 6.7/124 = 0.0540; Multiply 0.0540 by 100 to change
to percentage terms and get 5.4% as the rate of inflation between Year 1 and Year 2.
[text: E p. 146; MA p. 146]

23. What is “demand-pull” inflation?

“Demand-pull” inflation is the traditional view that inflation is caused by growing total
demand bidding up the prices of output in a period of nearly full or full employment.
[text: E pp. 147-148; MA pp. 147-148]

24. Inflation is frequently described as “too much money chasing too few goods.” Is this an
acceptable definition?

176
Technology, R&D, and Efficiency

This does not really define inflation completely. However, it does give anyone who has
a knowledge of the relationship between prices and money supply a good idea of the
demand-pull cause of inflation, and, therefore, an understanding of the concept. The
more money there is relative to the amount of goods and services available, the greater
will be the price level. This is an easy notion to grasp for most people, but it is not a
technical definition of inflation. [text: E pp. 147-148; MA pp. 147-148]

25. Describe cost-push inflation and its major source.

Cost-push or supply-side inflation is caused by the per unit cost of production rising so
that costs push up prices for firms to maintain profitability. A major source of this type
of inflation comes from so-called supply shocks. The supply shocks refer to situations
in resource markets such as the oil crises of 1973–1974 and 1979–1980 when imported
oil prices rose dramatically and pushed up energy costs for production and
transportation. [text: E pp. 148-149; MA pp. 148-149]

28. Explain the difference between real and nominal income. How can you get an
approximation of the percentage change in real income from one time period to another?

Real income is determined by dividing nominal income by the price level expressed in
hundredths. The percentage change in real income can be approximated by subtracting
the percentage change in the price level from the percentage change in nominal income.
[text: E pp. 149-150; MA pp. 149-150]

30. Assume that nominal income is $35,000 and the price index is 1.20 in year 1. In year 2,
nominal income rises to $38,000 and the price index rises to 1.25. What was the
percentage change in real income from year 1 to year 2? What was the absolute amount
of increase in real income? Make your calculations of the percentage change in real
income and the absolute change in real income using the approximation formula and
using the more precise method with index numbers.

First, calculate the percentage change by the approximation formula. Nominal income
has increased by 8.6% [($3,000/$35,000) x 100]. The price level has increased by 4.2%
[(.05/1.20) x 100]. The percentage change in real income was approximately 4.4%
(8.6% – 4.2%). To calculate the amount of the absolute increase in real income,
multiply 4.4% times $35,000, which equals $1,540.

The more precise calculation would use index numbers. Real income in year 1 was
$29,166.7 ($35,000/1.20). Real income in year 2 was $30,400 ($38,000/1.25). The
absolute change in real income was $1,233.3. The percentage change in real income
was 4.2% [($1,233/$29,166) x 100]. [text: E pp. 149-150; MA pp. 149-150]

31. Explain the difference between nominal and real interest rates.

Nominal interest rates are the stated rates of interest. The real rates are the nominal
rates corrected for the effects of inflation on the interest payments. The real rate
represents the increase in purchasing power that the lender receives from the borrower
and can be approximated by subtracting the rate of inflation from the nominal rate of
interest. [text: E pp. 151-152; MA pp. 151-152]

CHAPTER 9 Building the Aggregate Expenditures Model

177
Technology, R&D, and Efficiency

7. What are the marginal propensity to consume (MPC) and marginal propensity to save
(MPS)? How are the two concepts related? How are the two concepts related to the
consumption and saving functions?

The marginal propensity to consume is the ratio of a change in consumption to the


change in income which caused that change in consumption. The marginal propensity
to save is the ratio of the change in saving to the change in income which caused that
change in saving. The sum of the MPC and MPS for any change in disposable income
must always equal 1 because any fraction of a change in income which is not consumed
is saved. The MPC is the numerical value of the slope of the consumption schedule and
the MPS is the numerical value of the slope of the saving schedule. [text: E pp. 163-
164; MA pp. 163-164]

11. List four factors which could shift the consumption schedule.
Shifts in the consumption schedule could be caused by any of the nonincome
determinants of consumption and saving. This includes changes in any of the following:
wealth, expectations, personal taxes, and household debt. [text: E pp. 164-165; MA pp.
164-165]

12. What is the effect of increase in wealth on the consumption and saving schedules?

When wealth increases, it shifts the consumption schedule upward as people consume
more at each level of disposable income. There is an opposite effect on saving. The
saving schedule shifts downward at each level of disposable income because people
save less. [text: E pp. 164-165; MA pp. 164-165]

13. Other things being constant, what will be the effect of each of the following upon the
equilibrium level of GDP?
(a) An increase in the amount of liquid assets consumers are holding
(b) A sharp rise in stock prices
(c) A rapid upsurge in the rate of technological advance
(d) A sharp increase in the interest rate

(a) This should increase GDP because an increase in consumer wealth would lead to an
increase in consumer spending which would shift the aggregate expenditures curve
upward to a higher equilibrium output level.
(b) The probable effect of a sharp rise in stock prices would be to increase shareholder
purchases as a result of a rise in wealth, thus shifting the aggregate expenditures curve
upward and increasing the equilibrium level of GDP. It also could encourage business
investment with funds gained by issuing new shares of stock at the now higher prices.
This would also tend to increase GDP.
(c) This should increase GDP because of the impact on new investment spending and
possible increased consumer purchases of goods having the new technology. The
aggregate expenditures curve will shift upward and real quantity will rise.
(d) A sharp increase in the interest rate would limit consumer durables purchases and
also limit investment spending. Both of these events would cause a downward shift in
aggregate expenditures causing a decrease in GDP. One could even argue that higher
interest rates raise production costs and shift the aggregate supply curve leftward as
well, further leading to the GDP decline. [text: E pp. 164-166; MA pp. 164-166]

18. List five events that could cause a shift in the investment demand curve.

Five events would result from changes in the determinants of investment demand. For
example, changes in the price, cost of operation, or maintenance of particular investment
goods could cause the curve to shift; changes in business taxes favoring or penalizing
investment could cause it to shift; a technological change favoring new investment could

178
Technology, R&D, and Efficiency

cause a shift; changes in the stock of capital goods on hand will cause the existing
demand curve to shift; and changing expectations about future profits from investment
would have an effect. [text: E pp. 168-169; MA pp. 168-169]

19. What is the difference between the investment-demand curve and the investment
schedule for the economy?

The investment-demand curve shows the relationship between the real interest rate and
the level of investment spending. The relationship is an inverse one--the lower the
interest rate, the greater the investment spending--which means that the investment-
demand curve is downsloping. This curve can also be shifted by five factors that can
change the expected rate of return on investment.

The investment decisions of individual firms can be aggregated to construct an


investment schedule. It shows the amount business firms collectively intend to invest at
each possible level of GDP. A simplifying assumption is also made that investment is
independent of GDP, so the investment schedule is graphed as a horizontal line across
the possible levels of real GDP. [text: E pp. 167-170; MA pp. 167-170]

20. State four factors that explain why investment spending tends to be unstable.

Investment spending is based to a large extent on expectations about future profitability


and this can vary significantly from period to period. Technological changes affect
investment spending and these changes are not predictable in their timing. Investment
goods tend to be long lasting and “lumpy” in nature; that is, once a capital good is
purchased it lasts a long time and the expenditure will not be repeated on a frequent,
regular basis. Furthermore, this type of expenditure is usually large, so any changes
tend to be substantial on a firm-by-firm basis. Expectations and profits are both highly
variable. Actual profits may not meet expectations and this can affect expectations in
the future. Expectations are also based on many different external factors. [text: E pp.
170-172; MA pp. 170-172]

21. Compare the determinants of the consumption schedule and the investment schedule.
Most economists regard the investment schedule as being less stable than the
consumption schedule. Looking at the determinants of the two schedules, support this
contention.

The nonincome determinants of the consumption schedule are consumer wealth,


expectations, personal taxes, and household debt. The determinants of investment are
price of investment goods and their maintenance and operating costs, business taxes,
technological change, stock of capital goods on hand, and expectations. Comparing the
two lists there are some similarities. For example, both include expectations, related
price levels, and relevant taxes. However, the technological change and the stock of
capital goods on hand have no analogy in the consumption determinants.

These latter two determinants of investment support the contention of economists that
the investment schedule is more unstable than the consumption schedule. Technological
change is difficult to predict and certainly its impact would vary depending on the extent
of the change. The stock of capital goods on hand is a result of previous investment and
because of the nature of most capital goods, they can be made to last for a long period of
time. Once new capital spending occurs, it is “lumpy” in the sense that it will not be
repeated gradually, but only again when the particular capital good wears out or
becomes obsolete. Only the durable goods component of consumption is similar, but
most of consumer spending is of the more immediate type such as nondurable goods and
services which are primarily related to income and would not vary greatly from period
to period for most consumers.

179
Technology, R&D, and Efficiency

The basic determinant of consumption is the level of income, but nonincome factors
include wealth, the price level, expectations, household debt, and taxation. Aside from a
drastic change in government tax or transfer policies, the consumption schedule is quite
stable. That is, changes in disposable income are accompanied by predictable changes
in consumption spending. Furthermore the other factors are quite diverse and tend to be
self-canceling across the population.

The two basic factors determining the level of investment spending are the expected rate
return and the real interest rate. Since the former is based on expectations and the latter
based to a large extent on monetary policy, there is potential for wide variation. Add to
this the fact that investment goods are usually quite durable, and new investment can be
postponed depending on expectations, or once it is made there will be a period of time
before the new capital goods will need to be replaced. Also the fact that innovations
occur irregularly leads to the inability to plan for gradual investment in innovative
technology. Finally, actual current profits are often not as expected, so businesses can
be expected to shift their investment plans from year to year. [text: E pp. 164-172; MA
pp. 164-172]

22. Define the equilibrium level of output.

The equilibrium level of output is the level of output whose production will create total
income and spending which is just sufficient to purchase that output. [text: E pp. 172-
173; MA pp. 172-173]

23. Explain why saving equals planned investment at equilibrium GDP.

It is based on the fact that saving is income not consumed. Saving therefore represents a
“leakage” or diversion of potential spending from the income-expenditures stream.
Consumption falls short of total output by the amount of saving. However, investment
spending can be viewed as an “injection” into this income-expenditures stream. If
planned investment is equal to the amount of saving at a particular level of GDP, then
leakages equal injections and GDP will be in equilibrium. [text: E pp. 174-176; MA pp.
174-176]
24. Explain the difference between an equilibrium level of GDP and a level of GDP which
is in disequilibrium.

If GDP is not in equilibrium, then aggregate expenditures will exceed real GDP or vice
versa. If aggregate expenditures exceed real GDP, then businesses will find their
inventories reduced below the planned level of inventories. Businesses will therefore
expand production to replenish inventories and the economy will not be in equilibrium
until the level of planned inventories is met.

On the other hand, if real GDP exceeds aggregate expenditures, then business
inventories will be above the level planned. In order to bring inventories down to their
planned level, production and output will be reduced until the real GDP is equal to
planned aggregate expenditures. [text: E pp. 172-174; MA pp. 172-174]

28. Explain the difference between planned and actual investment in the economy. Why is
the distinction important?

Actual investment consists of both planned investment and changes in inventories.


Unplanned changes in inventories acts as a balancing item which equates the actual
amounts saved and invested in any period. At above equilibrium levels of GDP, saving
is greater than planned investment, and there will be an unplanned increase in
inventories. At below equilibrium levels of GDP, planned investment is greater than

180
Technology, R&D, and Efficiency

saving, and there will be an unplanned decrease in inventories. Equilibrium is achieved


when planned investment equals saving, and there are no unplanned changes in
inventories. [text: E pp. 174-176; MA pp. 174-176]

30. (Last Word) Explain Say’s law.

Say’s law, attributed to the 19th-century French economist, is that “supply creates its
own demand.” Essentially, this theory states that people engage in production in order
to be able to buy or demand other things. Whatever one earns from production, one
expects to spend on goods and services of equivalent value. Thus, supply (production)
has created its own demand. [text: E p. 177; MA p. 177]

32. (Last Word) What two events undermined the theory that supply creates its own
demand?

First, the Great Depression of the 1930s resulted in a sharp decline in production and
high rates of unemployment over a ten-year period. This event was inconsistent with
the classical theory that suggests that unemployment is only temporary. Second, John
Maynard Keynes developed an aggregate expenditures theory that countered Say’s law
and explained why unemployment and underspending can occur in an economy.
Keynes’ modern employment theory suggests that the macroeconomy was inherently
unstable and subject to fluctuations in output and employment because of the downward
inflexibility of wages and prices and lack of synchronization between investment and
saving decisions. [text: E p. 177; MA p. 177]

CHAPTER 10 Aggregate Expenditures: The Multiplier,


Net Exports, and Government

3. Define the multiplier. How is it related to real GDP and the initial change in spending?
How can the multiplier have a negative effect?

The multiplier is simply the ratio of the change in real GDP to the initial change in
spending. Multiplying the initial change in spending by the multiplier gives you the
amount of change in real GDP. The multiplier effect can work in a positive or a
negative direction. An initial increase in spending will result in a larger increase in real
GDP, and an initial decrease in spending will result in a larger decrease in real GDP.
[text: E p. 183; MA p. 183]

4. What are two key facts that serve as the rationale for the multiplier effect?

First, the economy has continuous flows of expenditures and income in which income
received by one person comes from money spent by another person who in turn receives
income from the spending of another person, and so forth. Second, any change in
income will cause both consumption and saving to vary in the same direction as the
initial change in income, and by a fraction of that change. The fraction of the change in
income that is spent is called the marginal propensity to consume (MPC). The fraction
of the change in income that is saved is called the marginal propensity to save (MPS).
The significance of the multiplier is that a small change in investment plans or
consumption-saving plans can trigger a much larger change in the equilibrium level of
GDP. [Text: E pp. 183-184; MA pp. 183-184]

5. Explain the economic impact of an increase in the multiplier.

181
Technology, R&D, and Efficiency

The multiplier magnifies the fluctuations in economic activity initiated by changes in


investment spending, net exports, government spending, or consumption spending. The
larger the multiplier the greater will be the impact of any changes in spending on real
GDP. [text: E pp. 183-184; MA pp. 183-184]

6. What is the relationship between the multiplier and the marginal propensities?

The multiplier is directly related to the marginal propensities. By definition, the


multiplier is related to the marginal propensity to save because it equals 1/MPS. Thus,
the multiplier and the MPS are inversely related. The multiplier is also related to the
marginal propensity to consume because it also equals 1/ (1–MPC). [text: E pp. 184-
186; MA pp. 184-186]

10. (Advanced analysis) Assume the consumption schedule for the economy is such that C
= 50 + 0.8Y. Assume further that investment and net exports are autonomous or
independent of the level of income and gross investment is 40 and net exports equal –10.
Recall that in equilibrium, Y = C + Ig + Xn.

(a) Calculate the equilibrium level of income for this economy.

(b) What will happen to equilibrium Y if gross investment falls to 20? What does this
tell us about the size of the multiplier?

(a) Equilibrium GDP is 400 = (50 = 0.8Y) + 40 + (–10)


(b) If gross investment falls by 20, GDP will fall by 100 because the multiplier is 1/.2
or 5 and 5 x 20 = 100 decline. The new equilibrium would be 300. [text: E pp. 182-
188; MA pp. 182-188]

11. What is the effect of net exports, either positive or negative, on equilibrium GDP?

Positive net exports increase aggregate expenditures beyond what they would be in a
closed economy and thus have an expansionary effect. The multiplier effect also is at
work. Positive net exports will lead to a positive change that is greater than the amount
of the initial change. Negative net exports decrease aggregate expenditures beyond
what they would be in a closed economy and thus have a contractionary effect. The
multiplier effect also is at work here. Negative net exports lead to a negative change in
equilibrium GDP that is greater than the initial change. [text: E pp. 187-188; MA pp.
187-188]

14. When international trade is considered, explain how net exports could be either positive
or negative additions to aggregate demand. In which case would the impact of net
exports be expansionary? Explain.
When exports exceed imports, net exports are a positive addition to aggregate demand.
When imports exceed exports, net exports are a negative addition to aggregate demand
because more money is being spent on products from other countries than foreigners are
spending on products made in the United States. Rather than adding to aggregate
demand this latter situation is a leakage from total expenditures. In the case where net
exports are positive and growing, their impact would be expansionary. [text: E pp. 186-
188; MA pp. 186-188]

15. How does the fact that imports vary directly with GDP affect the stability of the
domestic economy?

Actually this fact should help stabilize the domestic economy. During inflationary
periods of rapid growth, rising imports should dampen that growth in domestic
aggregate demand. During recessionary periods, the decline in imports should help to

182
Technology, R&D, and Efficiency

offset falling domestic demand as net exports should rise. In other words, a smaller M
makes the (X – M ) balance grow. [text: E pp. 186-188; MA pp. 186-188]

18. Explain the relationship between net exports and the following factors: prosperity
abroad, tariffs on American exports abroad, depreciation of the American dollar on
foreign exchange markets.

Prosperity abroad improves net exports because it means foreigners will buy more U.S.
exports.

Tariffs on American exports abroad will initially decrease net exports as it makes
American products more expensive to foreigners. (If the U.S. later retaliates with tariffs
of its own, the effect is less certain.)

Depreciation of the American dollar should lead to improved net exports as foreigners
will find the purchasing power of their money rising relative to goods priced in
American dollars. Conversely, Americans will find foreign goods more expensive in
terms of the dollars they need to exchange for foreign currency to buy foreign goods, so
exports rise and imports fall. [text: E pp. 188-189; MA pp. 188-189]

19. Describe the probable impact of an increase in government spending assuming no


change in taxes or private spending and less than full-employment output.

Assuming no change in taxes or private spending, the probable effect of an increase in


government spending will be expansionary. Furthermore, the government spending
increase will be multiplied in terms of its impact on equilibrium GDP. The simple
multiplier in this case should equal the reciprocal of the marginal propensity to save.
[text: E pp. 189-190; MA pp. 189-190]

20. Identify the relationship between GDP, taxes, and disposable income.

Disposable income consists partly of income earned by resources used in producing the
GDP minus the total taxes levied on productive incomes at the various production
stages. Depreciation allowances and corporate retained earnings are also deducted from
GDP and transfer payments are added to arrive at the figure for disposable income.
[text: E pp. 191-192; MA pp. 191-192]

21. “If taxes and government spending are increased by the same amount, there will still be
a positive effect on equilibrium GDP.” Explain.

The initial impact of government spending is to increase aggregate demand directly by


the amount of the increase in spending. Beyond that, spending is increased in
successive rounds of increased incomes that result by a fraction equal to the marginal
propensity to consume. This MPC-induced spending which results from the increased
government purchases will be exactly offset by the tax increase whose initial impact is
to reduce disposable income rather than aggregate demand directly. Thus, government
spending has an initial direct effect equal to the amount of the increase in G which will
not be offset. [text: E pp. 193-194; MA pp. 193-194]

22. Compare and contrast the recessionary gap and the inflationary gap.

A recessionary gap is the amount by which aggregate expenditures fall short of the
noninflationary full-employment level of GDP. Real GDP will be below full-
employment real GDP by a multiple amount of the recessionary gap. An inflationary
gap, on the other hand, is the amount by which aggregate expenditures exceeds the
noninflationary full-employment level of GDP. This gap will cause demand-pull

183
Technology, R&D, and Efficiency

inflation as nominal GDP rises to meet the higher level of aggregate expenditures, but
real GDP is already at its full-employment level. [text: E pp. 194-195; MA pp. 194-195]

23. If there is a recessionary gap of $100 billion and the MPC is 0.80, by how much must
taxes be reduced to eliminate the recessionary gap?

If the MPC is 0.80, then the MPS is 0.20 and the multiplier is equal to 5. Thus to reduce
a gap of $100 billion, taxes must be reduced by $25 billion, which is equivalent to
saying that disposable income rises by $25 billion. An increase in income of $25 billion
will cause an initial change in spending of $20 billion (or 0.8 × $25) and this multiplied
by 5 will result in an increase in GDP of $100 billion which is the amount of the
recessionary gap. [text: E pp. 194-196; MA pp. 194-196]

30. What happened to investment spending during the Great Depression? What factors
contributed to this situation?

A steep decline in investment spending was one of the major reasons the economy sank
into depression during the 1930s. Gross investment spending dropped 90 percent in real
terms.

The factors contributing to this decline were several. First, businesses had
overexpanded during the 1920s for the size of the market, which contributed to a rise in
business indebtedness. Businesses did not have the funds to spend much on new
investment. Second, there was also a decline in residential construction after the boom
of the early-to-mid 1920s. Residential construction is a component of investment
spending. Third, the stock market crash of 1929 deflated the extreme business optimism
and eliminated most interest in new capital investment. A fourth factor that contributed
to the Great Depression was a decline in the money supply. The tightening of monetary
policy by the Federal Reserve contributed to the reduction in investment spending
because it reduced aggregate expenditures. [text: E pp. 196-197; MA pp. 196-197]

31. What are four shortcomings of the aggregate expenditures model?

First, although the model can account for demand-pull inflation, it does not indicate how
much the price level will rise when aggregate expenditures are greater than the
productive capacity of the economy. Second, the aggregate expenditure model does not
explain why demand-pull inflation can occur before the economy reaches its full-
employment level of output. Third, the aggregate expenditures model does not explain
why the economy can expand beyond its full-employment level of output. Fourth, the
aggregate expenditures model does not account for cost-push inflation. [text: E p. 198;
MA p. 198]

CHAPTER 11 Aggregate Demand and Aggregate Supply

1. Why is there a need for an aggregate demand and aggregate supply model of the
economy? Why can’t the supply and demand model for a single product explain
developments in the economy?

The basic reason for an aggregate model is that there are thousands of individual
products in an economy. Single product supply and demand model does not explain:
(1) why prices in general rise or fall; (2) what determines the level of aggregate output;
and (3) what determines changes in the level of aggregate output. The aggregate model
is needed to explain these changes. It simplifies the analysis of prices by combining the
prices of all individual goods and services into one aggregate price level. It simplifies
the analysis of quantities by combining the equilibrium quantities of all individual goods

184
Technology, R&D, and Efficiency

and services into a singe entity called the real domestic output. [text: E pp. 203-204;
MA pp. 203-204]

3. What is the difference in the explanation of the shape of the aggregate demand curve
and a single product demand curve? After all, both demand curves show an inverse
relationship between price and quantity.

The aggregate demand curve shows an inverse relationship between the price level (the
general level of all prices) and real domestic output (the equilibrium quantity of all
products). The explanation of this inverse relationship is based on the real-balances
effect, the interest-rate effect, and the foreign-purchases effect. In this case, as the price
level rises, the quantity of real domestic output decreases.

The supply and demand model for a particular product shows an inverse relationship
between the price of that product and the quantity of that product. The explanation for
the inverse relationship between price and quantity in the demand for a single product is
based on the substitution and income effects. The substitution effect is not applicable to
the aggregate case because there is no substitute for all products in the economy. Also,
the income effect is not applicable to the aggregate case because income now varies
with aggregate output. [text: E pp. 203-204; MA pp. 203-205]

5. How can the aggregate demand curve be derived from the aggregate expenditures
model?

The aggregate demand curve is derived from the intersections of the aggregate-
expenditures curves and the 45-degree curve. As the price level falls, the aggregate
expenditures curve shifts upward and the equilibrium real GDP increases, but as the
price level rises, the aggregate expenditures curve shifts downward and the equilibrium
real GDP decreases. The inverse relationship between the price level and equilibrium
real GDP is the aggregate demand curve.

Note that for the aggregate-expenditure model that a decrease in the price level: (1)
increases the value of wealth; thus increasing the consumption schedule. A decrease in
the price level decreases the interest rate, thus increasing the investment schedule. A
decrease in the price level decreases imports and increases exports, thus increasing net
exports expenditures. Thus a decrease in the price level will increase aggregate
expenditures (and real domestic output) because of these changes in wealth, interest
rates, and net exports. These three effects are also the ones that define the inverse
relationship between the price level and real domestic output in the aggregate demand
curve. [text: E pp. 205-206; MA pp. 205-206]

6. Explain the relationship between the aggregate expenditures model in graph (A) below
and the aggregate demand-aggregate supply model in graph (B) below. In other words,
explain how points 1, 2, and 3 are related to points 1', 2', and 3'.

185
Technology, R&D, and Efficiency

Graph Graph
A A

Through the real-balances, interest-rate, and foreign purchases effects, the consumption,
investment, and net exports schedules and therefore the aggregate expenditures schedule
will rise when the price level declines and fall when the price level increases. If the
aggregate expenditure schedule is at (C + Ig + Xn)2 when the price level is P2, we can
combine that price level and the equilibrium output, GDP2, to determine one point (2')
on the aggregate demand curve. A lower price level such as P1 shifts aggregate
expenditures to (C + Ig + Xn)1, providing us with point 1' on the aggregate demand curve.
Similarly, a higher price level at P3 shifts aggregate expenditures down to (C + Ig + Xn)3
so P3 and GDP3 yield another point on the aggregate demand curve at 3'. [text: E pp.
205-206; MA pp. 205-206]

7. The determinants of aggregate demand “determine” the location of the aggregate


demand curve. Explain three of the four basic determinants of aggregate demand.
The four basic determinants of aggregate demand are found in Figure 11.1 in the text.
They are a change in consumer spending, a change in investment spending, a change in
government spending, and a change in net export spending. Explanations for each
follow.

A change in consumer spending could occur as a result of an increase or decrease in


consumer wealth resulting from factors such as increased stock values. It could also be
caused by a change in consumer expectations about the future, a change in the level of
household indebtedness, or a change in personal taxes.

Changes in investment spending may occur as a result of changes in the interest rate (not
related to changes in the price level), changes in profit expectations having to do with
predictions about future returns on possible projects, changes in business taxes,
technological improvements which induce more capital investment, and the degree of
existing excess capacity.

Government spending might change for any of a number of reasons, including a change
in priorities resulting from a change in elected officials.

Net export spending can change for two nonprice-level-related reasons such as rising or
falling national incomes in other countries and exchange rate changes unrelated to
domestic price levels. [text: E pp. 206-208; MA pp. 206-208]

10. Why does aggregate demand shift outward by a greater amount than the initial change in
spending?

The basic reason is because of the multiplier. As shown by the aggregate expenditures
model, an initial increase in spending times the multiplier will be the amount that

186
Technology, R&D, and Efficiency

aggregate expenditures shift upward. The size of this total increase will also be the
amount of the shift of the aggregate demand curve outward. Thus, the shift of the
aggregate demand curve will be equal to the initial change in spending times the
multiplier. [text: E p. 209; MA p. 209]

12. The illustration below shows the three ranges of the aggregate supply curve. Describe
why each range has the slope shown.

The horizontal range is flat because it presumes a level of GDP which is far below the
full-employment level of GDP. Therefore, additional resources can be employed
without bidding up prices because there is such a high degree of unemployed resources
and unused capacity. The intermediate range is upward sloping because at this level
GDP is approaching or at full employment. In some resource markets, full employment
will have been reached and prices will be bid up as producers seek to attract these
resources away from other uses. The vertical range is vertical because it presumes that
the economy is at full capacity. Thus, any increase in aggregate demand at this point
will simply result in higher prices, but no change in real GDP. [text: E pp. 209-211;
MA pp. 209-211]

187
Technology, R&D, and Efficiency

15. Describe the change in aggregate supply that should result from each of the following
changes in determinants. Assume that nothing else is changing besides the identified
change. (Use “Decrease” or “Increase.”) (a) A rise in the average price of inputs; (b)
An increase in worker productivity; (c) Government antipollution regulations become
stricter; (d) A new subsidy program is enacted for new business investment in
productive equipment; (e) Energy prices decline.

(a) Decrease; (b) Increase; (c) Decrease (unless the increase in antipollution device
production outweighs the decline in production caused by the increased cost of the
regulations); (d) Increase; (e) Increase [text: E pp. 211-213; MA pp. 211-213]

17. Prepare a list of government tax or spending policy options which would tend to shift
the aggregate supply curve rightward.

Tax cuts or rebates in any area which would improve productivity. For example, tax
credits for investment in new capital goods or buildings, tax credits for education and
training.

Tax breaks for various types of natural resource exploration and development especially
for resources important in energy and construction industries.

Reducing or eliminating tariffs on imported resources.

Tax penalties for companies which grant inflationary wage increases. [text: E pp. 211-
213; MA p. 211-213]

28. Is the downward price inflexibility applicable to today’s economy? Why or why not?

Some economists give many reasons for the downward price-level inflexibility in the
economy. They note that the price level has not declined since one year in the 1950s
despite the fact that there has been eight recessions since then. Also, there are a number
of reasons for price-level inflexibility that include long-term wage contracts, the
payment of efficiency wages, a minimum wage, the menu cost of making price changes,
and fear of price wars.

Other economists argue that downward price inflexibility is not applicable to today’s
economy because union power has diminished over the years and the 1981-1982
recession showed that wages could be cut in major industries. Also, foreign competition
reduces monopoly power in certain industries and keeps firms from resisting price cuts.
The past forces that caused price-level inflexibility have declined according to these
economists. [text: E pp. 217-218; MA pp. 217-218]

30. Some economists argue that it is easier to resolve demand-pull inflation than it is cost-
push inflation. Use the aggregate demand and aggregate supply model to explain this
assertion.

By shifting aggregate demand leftward when the equilibrium occurs in the intermediate
or classical range, the inflation rate should fall. This assumes that the price-level
increase was due to demand factors only. It is possible to decrease aggregate demand by
using tax policies which decrease consumer or business spending, or by using monetary
policies which tighten the availability of credit for spending.

However, the factors which cause leftward shifts in the aggregate supply schedule (see
Figure 11.6) are not as easy for government policy to control. Government cannot
quickly change worker demands for higher wages; it cannot quickly increase
productivity which would also bring down production costs; it cannot control the price

188
Technology, R&D, and Efficiency

of imported resources. These three factors have contributed much to cost-push inflation
in the past, and help to illustrate the difficulty in controlling cost-push inflation. [text: E
pp. 215-219; MA pp. 215-219]

31. Suppose an economic advisor to the President recommended a personal income tax
increase when the economy was operating in the intermediate range of the aggregate
supply curve. Indicate the expected effects on aggregate demand and on aggregate
supply.

The advisor would recommend this in hopes of dampening consumer demand and
lessening demand-pull inflation. This should happen as consumers find themselves with
less disposable income. However, at the same time there may be some less desirable
effects on aggregate supply. For example, workers might demand higher wages to
compensate for the higher taxes. Higher taxes may cause lessened work incentives
which could cause a decline in productivity and, therefore, a rise in production costs. In
other words, while raising personal taxes seems to be a correct policy for dampening
demand and demand-pull inflation, it may have an offsetting effect on supply which
could cause cost-push inflation. [text: E pp. 215-219; MA pp. 215-219]

CHAPTER 12 Fiscal Policy

4. Explain the effect of a discretionary cut in taxes of $40 billion on the economy when the
economy’s marginal propensity to consume is .75. By how much is output likely to
expand if the economy is operating in the horizontal range of its aggregate supply curve
and there are no complications to this fiscal policy? How does this discretionary fiscal
policy differ from a discretionary increase in government spending of $40 billion?

If MPC is .75, the multiplier is 4. A tax cut of $40 billion will result in initial increase
in consumption of $30 billion (.75 × $40 billion). This initial increase in spending will
ultimately result in an increase in consumption spending of $120 billion because of the
multiplier process. In contrast, an initial increase in government spending of $40 billion
will ultimately increase consumer spending by $160 billion (4 × $40) because none of
the initial increase is siphoned off as savings as would be the case with a $40 billion tax
cut. [text: E pp. 225-226; MA pp. 225-226]

6. Explain the aspects of expansionary and contractionary fiscal policy. During which
phases of the business cycle would each be appropriate?

Expansionary fiscal policy refers to increases in government spending or decreases in


taxes or both, so that the net effect on aggregate demand is an increase in net
government spending. Contractionary fiscal policy is the opposite: an increase in taxes
or decrease in government spending or both, so that the net effect on aggregate demand
is a decrease in net government spending.

Expansionary policy would most likely be used during a recession (or trough) phase. A
contractionary policy would most likely be employed near the peak of the business cycle
as the economy reaches full-employment GDP and the potential for inflation accelerates.
[text: E pp. 225-227; MA pp. 225-227]

7. Differentiate between discretionary fiscal policy and nondiscretionary or built-in


stabilization policy.

Discretionary fiscal policy is the deliberate manipulation of taxes and government


spending by the Congress to alter real domestic output and employment, to control
inflation, and to stimulate economic growth during a particular period of time.

189
Technology, R&D, and Efficiency

Nondiscretionary fiscal policy, on the other hand, is the change in government


expenditures or taxes which occurs automatically as a result of existing laws. In
particular, personal income taxes have progressive rates and will slow spending and
inflation as GDP expands; when GDP declines, taxes will decrease by a more than
proportionate amount allowing incomes and spending to decline at a slower rate than
GDP. There are also many transfer payment programs which become effective when
incomes decline or unemployment occurs to reduce the decline in disposable income.
Conversely, these programs automatically are reduced when the economy expands and
unemployment declines and spending increases. [text: E pp. 225, 229-230; MA pp. 225,
229-230]

8. Describe two ways the Federal government can finance a deficit and explain which
would have the more expansionary effect.

The government can borrow money from the private sector in which case it will be
competing with private business borrowers for funds. If planned investment spending is
“crowded out,” the impact of expansionary deficits will be offset by the decline in
investment spending.

The government can also finance a deficit by issuing new money which essentially
means that the Federal Reserve has financed the deficit. This type of financing would
be more expansionary than borrowing from the private sector. [text: E pp. 227-228; MA
pp. 227-228]

9. Describe two ways the Federal government could retire debt in the event of a budget
surplus and explain which would have the most contractionary impact.

The government could use a budget surplus to pay off existing debt which would
“recycle” funds back into the economy and potentially offset the decline in government
spending. Alternatively, the government could impound the surplus funds, or allow
them to stand idle, which means these funds are not injected into the economy and
would have a more contractionary effect than the first alternative. [text: E p. 228; MA
p. 228]

15. In Year 1, the full-employment budget showed a deficit of about $100 billion and the
actual budget showed a deficit of $150 billion one year. In Year 2, the full employment
budget showed a deficit of about $125 billion and the actual budget showed a deficit of
$150 billion. Based on these data, what can be concluded about the direction of fiscal
policy?

Fiscal policy was expansionary because the full-employment budget deficit increased
from one year to the next. The actual deficit is composed of the full-employment
portion and the cyclical portion. The full-employment portion of the actual budget
deficit rose from $100 to $150 billion. The cyclical portion is determined by taking the
actual deficit and subtracting the cyclical portion from it. The cyclical portion of the
actual deficit fell from $50 billion to $25 billion. The actual budget deficit did not
change, but it does not provide a good indication of the direction of fiscal policy. Only
the full-employment budget tells the direction of fiscal policy. [text: E pp. 230-232; MA
pp. 230-232]

16. What is the difference between the actual deficit, the full-employment deficit, and the
cyclical deficit?

The actual budget deficit for any year consists of the full-employment and the cyclical
deficit. The full-employment deficit is the difference between government expenditures
and tax collections which would occur if there were full employment output. The

190
Technology, R&D, and Efficiency

cyclical deficit is the portion of the actual deficit that arises because the economy is in
recession and is produced by this downturn in the business cycle. During a recession, a
cyclical deficit often occurs because tax revenues fall as incomes fall and government
expenditures increase as more is spent for government transfer payments and other
programs. The cyclical deficit occurs because of the operation of these automatic
stabilizers. [text: E pp. 230-231; MA p. 230-231]

19. In what fundamental way do the spending-taxation decisions of government differ from
the consumption-saving plans of households? Why is this difference significant?

The spending-taxation decisions of government are made in a political environment in


which the majority must be satisfied, or satisfied enough to continue to vote for its
elected representatives. Furthermore, since the government does not have a limited
lifespan and always has the ability to tax, deficit-spending and debt do not have the
same significance to governments that they do to individual households. Households
face a much more uncertain future with regard to their power to raise revenue (income)
and therefore must plan their spending and saving to coincide with their lifetime
earnings expectations.

The difference is significant because so many people try to draw an analogy between
government spending policies and household spending plans when it is usually not
appropriate to do so. [text: E pp. 233-234; MA pp. 233-234]

20. Identify the problems or complications that arise in the implementation of fiscal policy.

First there is a timing problem. Three lags are identified under the “timing problem”
category. There is a lag in recognizing the phase of the business cycle; there is an
administrative lag in deciding which policies to follow; there is an operational lag in
terms of the impact of policy once it is implemented.

Second, there is evidence of a political business cycle where particularly expansionary


policies are followed in election years whether or not economic conditions merit them.
Third, state and local fiscal policy may counteract federal fiscal policy.

Fourth, there is concern about possible offsetting effects of government borrowing


crowding out private spending that would occur in the absence of the government
deficit; and an offsetting net export effect which partly counteracts expansionary policy
or contractionary policy. [text: E pp. 233-235; MA pp. 233-235]

24. Comment on the statement: “Discretionary fiscal policy offers an ideal approach to
dealing with the nation’s economic problems.”

Discretionary fiscal policy does offer government policy makers potential tools
(changing taxes or government spending) to use for stimulating the economy during a
recession or for contracting the economy during a period of high inflation. Fiscal
policy, however, is not without its problems, criticisms, or complications. There are
timing problems in getting it implemented at the right time so it will be effective. There
are political problems in getting it accepted because it takes time to get the actions
passed through Congress and signed by the President. In addition, there are three major
criticisms of our complications with fiscal policy. Some economists are concerned that
expansionary fiscal policy which requires the Federal government to borrow money will
raise interest rates and crowd-out investment spending, thus reducing the expansionary
effect of the fiscal policy. A second complication arises from the connection of the
domestic economy to the world economy. Aggregate demand shocks from abroad or a
net export effect may increase or decrease the effectiveness of a given fiscal policy. The
third concern is whether an expansionary fiscal policy might actually increase the price

191
Technology, R&D, and Efficiency

level and have little effect on real output, especially if the economy is operating near its
full-employment level of output. [text: E pp. 233-236; MA pp. 233-236]

25. Explain how the net-export effect would reduce the effectiveness of fiscal policy.

If an expansionary fiscal policy brings with it higher interest rates, this could increase
the demand for American dollars by foreign investors seeking to earn the higher U.S.
returns. This appreciation of the dollar makes U.S. goods and services more expensive
to foreigners and foreign imports less expensive to Americans. The net export category
of aggregate demand will be reduced which would reduce the impact of expansionary
fiscal policy. A contractionary fiscal policy could have the opposite effect causing net
exports to increase that again reduces the desired effect of the contractionary fiscal
policy. [text: E pp. 235-237; MA pp. 235-237]

26. What fiscal policy is most likely to be invoked during a period of rapid inflation? A
period of severe unemployment? What political, investment, and international problems
might Congress encounter in enacting these policies and putting them into effect?

A contractionary fiscal policy would most likely be called for during rapid inflation,
especially if it seems to be demand-pull inflation. During severe unemployment, the
government would most likely initiate an expansionary fiscal policy.

Several problems are likely to arise in enacting either of these policies. Timing lags in
recognition, implementation, and impact are one concern. Another has to do with
political realities. A contractionary policy has many unpopular aspects to it because it
calls for raising taxes and for cutting government spending. There are also unique
problems associated with expansionary policy: crowding out is one potential result that
would reduce the expansionary effect of the policy. In both cases, the net-export part of
aggregate demand is likely to move in a direction that would tend to offset the policy.
[text: E pp. 233-237; MA pp. 233-237]

27. Contrast “supply-side” fiscal policy with “demand-side” fiscal policy.

“Supply-side” fiscal policy is the use of government tax or spending policies to alter the
supply schedule, that is, the production side of the economy. Tax reductions may have
an expansionary effect on aggregate supply as well as aggregate demand. Some
economists argue that as taxes are cut, savings and investment will increase. Also tax
cuts could be aimed specifically at encouraging business investment such as investment
tax credits. “Supply-side” economists also argue that tax increases can reduce tax
revenues rather than increasing them due to a leftward shift in the aggregate supply
curve which causes a decrease in domestic output. Chapter 17 addresses these theories
in more detail. [text: E pp. 236-237; MA pp. 236-237]

29. Using the aggregate demand-aggregate supply diagram shown below, draw in relevant
curves to indicate potential supply-side effects of a tax decrease. Explain the outcome
of your illustrated shift in terms of the new equilibrium quantity and price level.

192
Technology, R&D, and Efficiency

The potential effects on the supply side of a tax decrease would be to shift the supply
curve downward and rightward. This would be more likely to occur in the case of a tax
decrease targeted for business taxes or providing investment tax credits, increased
depreciation allowances, and other such tax breaks for business. See graph below.
[text: E pp. 236-238; MA pp. 236-238]

CHAPTER 13 Money and Banking

2. Money is what money does. Explain.

This refers to the idea that money (at least paper money and checks) has no intrinsic
value. It is valuable only in terms of its acceptability in exchange for goods and
services. In other words, it is valuable only in terms of what it does: act as a medium of
exchange, a unit of account, and a store of value. [text: E pp. 244-245; MA pp. 244-
245]

3. Some government bonds can be redeemed for currency or a check at banks. Why, then,
isn’t it universally agreed that government bonds are part of the money supply?

The question literally answers itself. These assets must be exchanged for currency or a
check (which is money by definition) before they are generally acceptable in exchange
for goods and services. While they can be exchanged for currency or a check, bonds are
one step removed from being spendable as money. [text: E pp. 245-246; MA pp. 245-
246]

4. What is the difference between the M1 and M2 definitions of the money supply?

Both M1 and M2 are definitions of the economy’s money supply. M1 is the definition
of the money supply with the highest degree of liquidity, the money supply used mainly
for transactions purposes. M1 consists of currency (coins and paper money) and
checkable deposits. M2 consists of everything in M1 plus noncheckable savings
deposits, small time deposits, money-market deposit accounts and money- market
mutual fund balances. M2 is a broader, but less liquid, definition of the money supply.
It includes everything in M2 plus large time deposits ($100,000 or more). M2 and M3
are considered near monies because they do not function directly as a medium of
exchange, but can be converted to currency or checkable deposits. [text: E pp. 245-247;
MA pp. 245-247]

193
Technology, R&D, and Efficiency

8. Are currency and checkable deposits owned by the government (U.S. Treasury) and the
Federal Reserve Bank, commercial banks, and other financial institutions part of the
money supply? Explain.

No. Currency and checkable deposits at these institutions are not counted. In the case
of commercial banks and other financial institutions, if currency from an individual is
deposited in a checking account, and both the currency and checkable deposit were then
counted as part of the money supply, then it would be double counting. So, the money
supply consists only of the currency and checkable deposits held by individuals or
businesses at financial institutions. The exclusion of currency and checkable deposits
held by government is more arbitrary, but permits economists to focus on the money
supply in the private sector of the economy. [text: E pp. 246-247; MA pp. 246-247]

9. Explain the difference between a money-market deposit account and a money-market


mutual fund.

A money market mutual fund is offered through a financial investment company and
investors buy shares in the fund. Such funds invest in short-term (less than one year)
credit instruments such as Treasury bills and short-term certificates of deposit that are
known as money-market instruments. The money-market deposit account is a type of
savings account offered by banks and thrifts that pool individual deposits to buy a
variety of short-term securities. These accounts have minimum balance requirements
and limit how often money can be withdrawn. Because they are bank accounts,
however, they are insured by the relevant deposit insurance fund. [text: E p. 247; MA p.
247]

10. Credit cards are not really money. Explain.

Credit cards represent the ability to get an instant loan that can be exchanged for goods
or services. However, that loan must be paid at some point in the future with money.
[text: E p. 248; MA p. 248]

11. Why is money considered to be debt?

The major parts of the money supply are currency and checkable deposits. These items
are debts, or promises to pay. Paper money is the circulating debt of the Federal
Reserve banks. Checkable deposits are debts of commercial banks and thrifts. Neither
the currency nor checkable deposits have any intrinsic value. They are simply
circulating paper to which people must attach value. [text: E p. 248; MA p. 248]

12. Discuss three major points about what gives money its value.

First, currency and demand deposits (M1 definition) are considered money because
these items are accepted as payment for goods and services. Money must be acceptable
to serve its function as a medium of exchange. Second, the government mandates
through law that paper money be accepted as payment for debts. While checks are not
mandated by law as money, government agencies do back demand deposits at banks
with deposit insurance that helps to maintain the acceptability of this form of money.
Third, money is relatively scarce. There is a reasonably constant demand for money for
transactions purposes and future uses. The supply of money will determine the value or
“purchasing power” of each unit of money. The supply of money is controlled by

194
Technology, R&D, and Efficiency

monetary institutions (the Federal Reserve System) that attempt to maintain a


reasonably stable purchasing power for money. [text: E pp. 249-250; MA pp. 249-250]

13. What are the two reasons that people want to hold money? In other words, what are the
two types of demand for money?

People want to hold money for transactions purposes and as a form of liquid assets.
Thus, economists talk about a transactions demand for money and an asset demand for
money. [text: E pp. 251-252; MA pp. 251-252]

20. Analyze what would happen to the equilibrium rate of interest in the money market if
the supply of money were increased while the demand schedule remained the same.

Because the quantity of money demanded is inversely related to the interest rate, an
increase in the supply of money would result in a higher equilibrium quantity of money
being demanded at a lower equilibrium rate of interest. [text: E pp. 253-254; MA pp.
253-254]

23. Answer the next two questions using the following information: The price of a bond
with no expiration date is $1000 and its fixed annual interest payment is $50; bond
annual rate of interest is 5%.

(a) If the price of this bond decreases by $250 to $750, what will its effective interest
rate be for the new buyer?
(b) If the price of this bond increases to $1200, what will its effective interest rate be for
the new buyer?
(a) $50/$750 = 1/15 = 6.67%
(b) $50/$1200 = 1/24 = 4.17% [text: E pp. 253-254; MA pp. 253-254]

26. Describe the major parts of the Federal Reserve System.

First, the Federal Reserve System is overseen by the Board of Governors. This Board is
responsible for control of the supply of money and the banking system. The President
appoints the seven members of the Board. Second, several important bodies help the
Board establish and conduct policy — the Federal Open Market Committee (FOMC),
which establishes policy over the buying and selling of government securities, and three
Advisory Councils that provide inputs from commercial banks, thrift institutions, and
consumer-related organizations. Third, there are twelve regional Federal Reserve
Banks. They serve as central banks, quasi-public banks, and banker’s banks. [text: E
pp. 254-256; MA pp. 254-256]

29. The Federal Reserve Banks are bankers’ banks. Explain.

This means that the Federal Reserve Banks perform essentially the same functions for
banks as the banks perform for the public. Just as banks and thrifts accept deposits of
and make loans to the public, so the Federal Reserve Banks accept deposits and make
loans to banks and thrifts. [text: E p. 256; MA p. 256]

30. What are the seven functions of the Federal Reserve System? Which one is most
important?

The seven functions are: (1) issuing currency; (2) setting reserve requirements and
holding required reserves of banks and thrift institutions; (3) lending money to banks
and thrifts; (4) collecting and clearing checks for banks and thrifts; (5) serving as the
fiscal agent for the U.S. government; (6) supervising the operation of member banks;

195
Technology, R&D, and Efficiency

and (7) controlling the money supply. Controlling the money supply to meet the needs
of the economy is the most important function. [text: E p. 257; MA p. 257]

33. Cite five recent developments that have affected money and banking in the United
States.

First, there has been a relative decline in the number of banks and thrifts. Second, there
has been consolidation among banks and thrifts because of mergers. Third, the services
offered by financial institutions are becoming similar because of fewer legal restrictions
on activity by type of institution. Fourth, financial markets are now more globalized and
integrated because of advances in computer and communications technology. Fifth, the
character of money has changed with the shift to the use of electronic money and other
forms of payment. [text: E pp. 258-260; MA pp. 258-260]

CHAPTER 14 How Banks and Thrifts Create Money

1. Describe the basic features of a commercial bank’s balance sheet.

On the left side of the balance sheet of a commercial bank is a statement of the bank’s
assets. On the right side of the balance sheet are the claims of the owners of the bank,
called net worth, and claims of the nonowners, called liabilities. This relationship would
be written in equation form as: assets = liabilities + net worth. [text: E p. 265; MA p.
265]

2. What are the major assets and the major claims (liabilities) on a commercial bank’s
balance sheet?

Major assets include reserves, loans, and government securities. Major claims are
customer deposits. In Chapter 14, the only major assets mentioned were reserves and
loans. [text: E p. 265; MA p. 265]

3. What is the history behind the idea of a fractional reserve banking system?

Early traders used gold in making transactions. They realized that it was inconvenient
and not safe to move gold for every transaction. So they deposited their gold with a
goldsmith. The goldsmiths had safes for gold and precious metals, which they often
kept for consumers and merchants. They issued receipts for these deposits. Receipts
came to be used as money in place of gold because of their convenience. The
goldsmiths became aware that the stored gold was never fully redeemed. The goldsmiths
realized they could “loan” gold by issuing paper receipts to borrowers, who agreed to
pay back gold plus interest. Such loans originated “fractional reserve banking,” because
the actual gold in the vaults became only a fraction of the receipts held by borrowers and
owners of gold. [text: E pp. 265-266, MA pp. 265-266]

4. What are the two significant characteristics of the fractional reserve banking system?

The two significant characteristics are: (1) banks can create money by lending more
than the original reserves on hand. The smaller the amount of reserves viewed as
necessary, the larger the amount of money that could be created. (2) The lending
policies of fractional reserve banks must be prudent to prevent bank “panics” or “runs”
by depositors worried about their funds on deposits at the institutions. The reserves
must be able to cover the bank “runs.” [text: E p. 266, MA p. 266]

5. Why are financial institutions required to keep reserves?

196
Technology, R&D, and Efficiency

Reserves are required to constrain the amount of bank lending (money creation) that can
occur. Without required reserves, theoretically banks would have unlimited power to
lend and thereby, expand the money supply without any limit. [text: E p. 266; MA p.
266]

7. What happens to the money supply when a bank accepts deposits of currency from the
public and places it in checkable deposits (or checking accounts)?
The composition of the money supply changes, but there has been no change in the
economy’s total supply of money. In this case, bank money or checkable deposits have
increased and currency held by the public has decreased by an equal amount. [text: E p.
267; MA 267]

8. Arrange the following items in the form of a commercial bank’s balance sheet, and
explain how each might come into being.

Capital stock, $300,000; Reserves, $60,000; Property, $290,000; Checkable deposits,


$150,000; Securities, $40,000; Loans, $60,000

Assets Liabilities
Reserves $ 60,000 Checkable Deposits $150,000
Loans 60,000 Capital Stock 300,000
Securities 40,000
Property 290,000

(a) Reserves could come from deposits or cash capital of owners.


(b) Loans represent what the bank has temporarily given to borrowers who must repay
the money at a later date.
(c) Securities represent government notes which the bank has purchased and which can
be redeemed for money.
(d) Property represents the real and personal property owned by the bank such as the
building, equipment, and so forth.
(e) Checkable deposits are customers’ checkable deposits.
(f) Capital stock is the value of the owners’ equity. [text: E pp. 266-267; MA pp. 266-
267]

9. Define the reserve ratio.

The required reserve ratio is the fraction or percentage of reserves which must be held
against deposits by financial institutions regulated by the Federal Reserve Board of
Governors. The reserve requirements are given in the text in Table 14.1, p. 267. [text:
E p. 267; MA p. 267]

10. Give an equation that shows the relationship between actual, required, and excess
reserves.

Actual reserves = required plus excess reserves; or alternatively, excess reserves =


actual minus required reserves. [text: E p. 268; MA p. 268]

14. When a check is drawn against bank A and deposited in another bank, the first bank
loses reserves as the check is cleared. Yet the check collection involves no loss of
reserves by the banking system. Explain what significance this has for the lending
ability of the system as a whole.

The system does not lose reserves as long as checks are being redeposited in other
banks. The reserves simply move from one bank to another within the system. This
means that the reserves are still available within the system to support loans and since

197
Technology, R&D, and Efficiency

reserves must be only a fraction of deposits, they can support a multiple of the reserve
amount in terms of loan and deposit values. Reserves are sometimes called high-
powered for that reason. [text: E pp. 269-272; MA pp. 269-272]

16. What is the effect on the money supply when a commercial bank buys government
securities from the public?

The effect is the same as bank lending. The bank buying the securities issues a check
that in turn gets deposited in a bank. The checkable deposits of that bank have increased
its lending ability by increasing its excess reserves. [text: E p. 273; MA p. 274] {Is MA
274 correct?}

18. What are the two conflicting goals of bankers? How do these conflicting goals get
resolved in the federal funds market?

First, banks are in business to make a profit just as are other businesses. They earn
profits primarily on loans and by buying and selling securities. Second, banks must
seek safety by having liquidity to meet the cash needs of depositors and to meet
transactions as checks clear. Banks can borrow from one another to meet short-term
needs for cash or reserves in the federal funds market. In this market banks borrow
available reserves from other banks on an overnight basis. The rate paid is called the
federal funds rate. [text: E p. 273, MA p. 273]

19. What is meant by the “Federal funds market” and what is the Federal funds rate?

When financial institutions find themselves temporarily short of reserves, they can
borrow from other institutions’ reserves on an overnight or very short-term basis. The
supply and demand for such reserve funds is known as the Federal funds market and the
rate at which these funds are borrowed is the Federal funds rate. [text: E p. 273; MA p.
273]

20. Answer the next question based on the following consolidated balance sheet for the
commercial banking system. Assume the required reserve ratio is 30 percent. All
figures are in millions of dollars.

Assets Liabilities
Reserves $200 Deposits $600
Securities 500 Capital Stock 700
Loans 100
Property 500

(a) What is the amount of excess reserves in this commercial banking system?
(b) What is the maximum amount that the money supply can be expanded?
(c) If the reserve ratio fell to 25 percent, what is now the maximum amount that the
money supply can be expanded?

(a) Required reserves are $600 million x .30 = $180 million. Actual reserves are $200
million, so excess reserves are $20 million.
(b) The monetary multiplier is 1/.3 or 3.33. Maximum expansion of the money supply
is $20 million x 3.33, or 66.67 million.
(c) If the reserve ratio was 25%, then excess reserves would be $50 million [$200
million – (.25 x $600 million)]. The monetary multiplier would be 1/.25 or 4, so the
maximum expansion of the money supply is $200 million [4 x $50 million]. [text: E pp.
274-277; E pp. 274-277]

198
Technology, R&D, and Efficiency

21. If the balance sheet above were for the entire banking system instead of just a single
bank, by how much could loans be expanded?

The system as a whole could support up to $30,000 in new loans because (assuming no
leakages) the $10,000 in excess reserves would be in the system as if the system were
one gigantic bank. As long as those reserves are in the system they must equal 33% of
new loans. $10,000 is 33% of $30,000. Therefore, $30,000 worth of new loans can be
created in the system with $10,000 of excess reserves. (The money multiplier is 1/.333
or 3; 3 times $10,000 = $30,000.) Checkable deposits would then become $180,000 and
actual reserves of $60,000 would just meet the legal requirement. [text: E pp. 274-277;
MA pp. 274-277]

199
Technology, R&D, and Efficiency

22. Define the monetary multiplier.

Because banks need only keep a fraction of their deposits in reserve, the deposit
multiplier is the multiple found by the ratio of the potential amount of deposits that can
be supported by a given amount of reserves. The multiplier, m, is the reciprocal of the
required reserve ratio R. (m = 1/R). For example, if the required reserve ratio is 10%,
then m = 1/.10 = 10. [text: E p. 277; MA p. 277]

24. Identify two types of “leakages” which could reduce the money multiplier.

Required reserves are one type of obvious leakage from the potential lending ability of
financial institutions since this is money they must keep. Other “leakages” include
currency held outside of banks and excess reserves held by banks and other financial
institutions. [text: E pp. 277-278; MA pp. 277-278]

25. What is the rationale for government control of the money supply?

The actions of banks reinforce the negative cyclical pressures in the economy. The
pressures must be counteracted by a government institution to achieve greater economic
stability. In prosperous times, banks extend credit to the maximum because there is
little risk of default by borrowers. These banking actions increase aggregate demand
and contribute to inflationary pressure in the economy. Conversely, during a
recessionary period, banks may be unwilling to make loans or extend credit, reinforcing
contractionary tendencies in the economy. A monetary institution such as the Federal
Reserve tries to achieve economic stability and promote economic growth by reducing
the lending capacity of banks during prosperous times to an amount consistent with the
needs of the economy. During a recessionary period, the monetary institution would
encourage bank lending to help expand the economy. [text: E p. 278; MA p. 278]

CHAPTER 15 Monetary Policy

1. What is the goal of monetary policy?

The goal of monetary policy is to stabilize the economy to promote price-level stability,
full employment and economic growth. [text: E p. 282; MA p. 282]

2. Identify the major items in the consolidated balance sheet of the Federal Reserve Banks.

The Fed’s balance sheet contains two major assets: (1) securities, which are federal
government bonds purchased by Fed; and, (2) loans to banks. The balance sheet
contains three major liabilities: (1) reserves of banks held as deposits at Federal Reserve
Banks; (2) U.S. Treasury deposits of tax receipts and borrowed funds; and (3) Federal
Reserve Notes outstanding, our paper currency. [text: E pp. 282-283; MA pp. 282-283]

4. What are the three principal tools of monetary policy? Explain how they can be used.

The Federal Reserve Banks use three principal tools (techniques or instruments) to
control the reserves of banks and the size of the money supply. (1) The Federal Reserve
can buy or sell government securities in the open market to change the lending ability of
the banking system: (a) buying government securities in the open market from either
banks or the public increases the excess reserves of banks; (b) selling government
securities in the open market to either banks or the public decreases the excess reserves
of banks. (2) The Fed can raise or lower the reserve ratio: (a) raising the reserve ratio
decreases the excess reserves of banks and the size of the monetary (checkable-deposit)
multiplier; (b) lowering the reserve ratio increases the excess reserves of banks and the

200
Technology, R&D, and Efficiency

size of the monetary multiplier. (3) The Fed can also raise or lower the discount rate:
(a) raising the discount rate discourages banks from borrowing reserves from the Fed;
(b) lowering the discount rate encourages banks to borrow from the Fed. [text: E pp.
284-289; MA pp. 284-289]

5. What is the difference between the Federal Reserve Banks’ purchases of securities from
the commercial banking system and those from the public? Give an example.

Assume that the commercial banks are “loaned up.” Purchases of bonds by the Fed
from commercial banks increase actual reserves and excess reserves of the commercial
banks by the full amount of the bond purchase. Purchases of bonds by the Fed from the
public increase actual reserves, but also increase checkable deposits. Some of the
checkable deposits must be kept as legal reserves, so the commercial banking system
has fewer excess reserves to lend out. Despite this difference the end result is the same
amount of increase in the money supply.

For example, if the Fed buys a $1,000 bond from commercial banks, the banks have
$1,000 in excess reserves to lend. If the reserve ratio is 20 percent, then the commercial
banks can increase the money supply by $5,000. If the Fed buys a $1,000 bond from the
public, then $1,000 in checkable deposits is created. The bank can lend the excess
reserves, which in this case will be $800 because 20 percent of $1,000 must be kept as
legal reserves. The $800 in excess reserves increase the money supply by $4,000.
Adding this $4,000 in bank lending to the $1,000 in new checkable deposits results in a
total increase in the money supply of $5,000. [text: E pp. 284-287; pp. 284-287]
6. Both Federal Reserve Banks and commercial banks buy and sell government securities,
but for substantially different reasons. Explain.

The Federal Reserve Banks buy and sell securities with the macroeconomy in mind.
They are pursuing either an easy or tight money policy when they buy or sell securities.
However, commercial banks buy and sell securities in order to improve their individual
bank’s profitability.

Securities are liquid assets which pay interest, and therefore are attractive investments
for banks to obtain with their idle reserves. If their cash reserves fall, they can easily
sell securities to obtain the needed reserves. [text: E pp. 284-287; MA pp. 284-287]

7. Explain how a change in the reserve ratio affects the money supply.

An increase in the reserve ratio will decrease the size of the monetary multiplier and
decrease the excess reserves held by commercial banks, thus causing the money supply
to decrease. A decrease in the reserve ratio will increase the size of the monetary
multiplier and increase the excess reserves held by commercial banks, thus causing the
money supply to increase. [text: E pp. 287-288; MA pp. 287-288]

8. Differentiate between easy and tight monetary policies.

An easy monetary policy is where the Federal Reserve attempts to expand the money
supply to stimulate aggregate expenditures in order to increase employment and output.
Buying securities, reducing the reserve ratio, and lowering the discount rate are the
appropriate directional changes that lead to an expanded money supply. A tight
monetary policy is the opposite. It is where the Federal Reserve attempts to reduce the
money supply to dampen spending and inflation. Selling securities, raising the reserve
ratio, and raising the discount rate are the appropriate changes leading to a reduced
supply of money. [text: E p. 289; MA p. 289]

9. Which tool of monetary policy is most important? Why?

201
Technology, R&D, and Efficiency

Open-market operations are the most important tool of monetary policy. Changes in the
discount rate are less effective because bank reserves are relatively small and require
action by commercial banks. Reserve requirements are rarely changed. Reserves do not
earn interest so an increase in reserve requirements would be costly to banks, making
this policy move less attractive. Open-market operations are used most often because
they are very flexible and have an immediate effect on bank reserves. [text: E pp. 289-
290; MA pp. 289-290]

10. Explain the impact of each of the following upon commercial bank reserves: (a) the
Federal Reserve sells government bonds in the open market to private buyers; (b) the
commercial banks reduce their indebtedness to the Federal Reserve Banks; (c) the
Treasury makes a number of large disbursements in accelerating space research.

(a) Selling bonds to private buyers would decrease bank reserves as individuals
withdraw money from deposits by writing checks on them or by physically withdrawing
money to pay for the bonds.
(b) If the commercial banks reduce their indebtedness to the Federal Reserve Banks,
this should decrease reserves as they use their reserves to pay back the debts to the Fed.
(c) Assuming that the Treasury is making these disbursements with new money and not
by simultaneously selling bonds, this should increase the bank reserves as the recipients
deposit the funds received from the space research programs. [text: E pp. 284-289; MA
pp. 284-289]

14. How does monetary policy affect equilibrium GDP? How can it address the problem of
recession or slow growth? Inflation?

Monetary policy affects the equilibrium GDP in many ways. In the money market, the
demand curve for money and the supply curve of money determine the real interest rate.
This rate of interest in turn determines investment spending. Investment spending then
affects aggregate demand and the equilibrium levels of real output and prices. If
recession or slow economic growth is a major problem, the Federal Reserve can institute
an easy money policy that increases the money supply, causing the interest rate to fall
and investment spending to increase, thereby increasing aggregate demand and
increasing real GDP by a multiple of the increase in investment. If inflation is the
problem, the Federal Reserve can adopt a tight money policy that decreases the money
supply, causing the interest rate to rise and investment spending to decrease, thereby
reducing aggregate demand and inflation. [text: E pp. 291-294; MA pp. 291-294]

15. Other things being equal, what effect will each of the following have on the equilibrium
rate of interest? (a) an increase in the supply of money; (b) an increase in the
equilibrium level of national income; (c) a decrease in the supply of money; (d) a
leftward shift of the asset demand for money.

The equilibrium rate of interest should increase in b and c; should decrease in a and d.
[text: E pp. 290-292; MA pp. 290-292]

202
Technology, R&D, and Efficiency

16. Use the below graphs to answer the following questions assuming the nominal GDP in
the economy is given.

(a) Look at graph A and suppose the supply of money increases from 100 to 200. What
will be the equilibrium rate of interest?
(b) Look at graph B which shows an investment-demand curve for this economy.
Given the answer to part (a) above, how much will investors plan to spend on capital
goods?
(c) What will happen to aggregate demand?
(d) Now trace what will happen in parts (a)–(c) if the money supply increases to $300.

(a) 6%
(b) $120
(c) Aggregate demand will increase by a multiple of the increase in investment
depending on the size of the multiplier.
(d) 3%; $170; Aggregate demand will increase by a multiple of the decrease in
investment depending on the size of the multiplier. [text: E pp. 290-292; MA pp. 290-
292]

19. Suppose the economy is experiencing a recession and high unemployment. What would
be the mainstream interpretation of how an easy money policy would address this
problem?

With an easy money policy, the Federal Reserve buys bonds, lowers the reserve ratio, or
lowers the discount rate. As a consequence of these actions, excess reserves increase,
which in turn increases the money supply. When this happens, interest rates fall,
investment spending increases and aggregate demand increases. The end result is a rise
in real GDP by a multiple of the increase in investment. [text: E pp. 292-293; MA pp.
292-293]

20. Suppose the economy is experiencing inflation. What would be the mainstream
interpretation of how a tight money policy would address this problem?

203
Technology, R&D, and Efficiency

With a tight money policy, the Federal Reserve sells bonds, raises the reserve ratio, or
raises the discount rate. As a consequence of these actions, excess reserves decrease,
which in turn decreases the money supply. When this happens, interest rates rise,
investment spending decreases and aggregate demand decreases. The end result is a fall
in real GDP by a multiple of the decrease in investment. [text: E pp. 293-294; MA pp.
293-294]

21. How is monetary policy affected by aggregate supply?

Monetary policy primarily affects investment spending and aggregate demand, real
output, and the price level. The aggregate supply curve, however, will determine
whether the shift in aggregate demand primarily increases output or the price level. For
example, an easy money policy will cause an increase in the aggregate demand. If this
increase occurs in the horizontal portion of the aggregate supply curve, then monetary
policy will have a positive effect on real output. If, however, the change in monetary
policy results in an increase in aggregate demand in the vertical portion of the aggregate
supply curve, the primary result will be an increase in the price level. An increase in
aggregate demand in the intermediate portion of the aggregate supply curve will
increase both real output and the price level but some of the increase will be lost on an
increase in the price level. [text: E p. 294; MA p. 294]

22. List three strengths of monetary policy for achieving economic stability.

Monetary policy is relatively speedy and flexible relative to fiscal policy because the
decision-making body is smaller and the decisions to change monetary policy can be
implemented immediately. A second strength is that monetary policy is largely
removed from political pressure since the members of the Board of Governors are
appointed to 14-year terms. Unpopular, but necessary, changes can thus be made which
might not be possible with fiscal policy where the decision makers are elected officials
who may be reluctant to make unpopular decisions. Third, there are many economists
who believe that monetary policy is the key determinant to the level of economic
activity and that fiscal policy is weak or ineffective in the absence of a coordinated
monetary policy. Monetary policy was successfully used in the 1980s and 1990s. [text:
E pp. 294-295; MA pp. 294-295]
23. Discuss the relative merits of monetary policy under conditions of demand-pull inflation
or recession.

Monetary policy may have the upper hand in reducing demand-pull inflation. The
Federal Reserve can conduct open-market operations relatively swiftly. The tough
decision-making is also isolated from political pressure. Monetary policy has also been
successful in reducing inflation. Also, monetary policy can stimulate an economy
during recession. But there are limits in this situation. An easy money policy may not
be effective if business expectations have fallen to the point that low interest rates will
not induce firms to borrow and purchase new capital. [text: E pp. 294-296; MA pp.
294-296]

24. What is the velocity problem for monetary policy during inflation? During recession?

In an inflationary period, the velocity of spending may increase to offset a tight money
policy because people may decide to hold less money in a period of rising prices and
interest rates. In a recession, an easy monetary policy also has a velocity problem.
There is the possibility that velocity will decline in a recession offsetting in part the
intent of the easy money policy. [text: E p. 295; MA p. 295]

25. Explain what is meant by cyclical asymmetry with regard to monetary policy effects.

204
Technology, R&D, and Efficiency

Cyclical asymmetry refers to the observation that a tight monetary policy seems to
achieve its objective of reducing aggregate demand much more effectively and
consistently than an easy monetary policy is able to achieve its objective of increasing
aggregate demand. During recession an expanded money supply and low interest rates
may not be enough to encourage more borrowing and spending if investors are
pessimistic about the future and lenders are cautious about lending. [text: E pp. 295-
296; MA pp. 295-296]

26. What are the political and economic limitations upon (a) fiscal policy and (b) monetary
policy?

Political limitations on fiscal policy are the strongest because tax and spending policies
are designed and ratified by Congress whose members are up for re-election every few
years. Thus, it is difficult for Congress to enact cuts in spending or hikes in taxes which
would be politically unpopular. Political limitations on monetary policy are less strong
since the members of the Board of Governors are appointed to 14-year terms and are
thus largely removed from political pressures. However, Congress does have the
ultimate power to change the system and the members of the board, so they cannot be
entirely oblivious to political impact of their decisions.

The economic limitations are closely related to the political, but would include the
ability and willingness of the public to pay increased taxes or withstand cuts in
government programs or to withstand the impact of a tight monetary policy. In other
words, policies to fight inflation are limited by the severity of their impact on incomes
and employment.

The economic limitations on expansionary policies are related to predictions about the
future impact of such policies. In other words expansionary policies can overcorrect
problems of a recession and create inflationary pressures in the future. [text: E pp. 295-
296; MA pp. 295-296]

205
Technology, R&D, and Efficiency

27. What has been the recent focus of monetary policy?

The Federal Reserve has been targeting the Federal funds rate in recent years. For
example, to stimulate the economy, the Fed can reduce the Federal funds rate. It
achieves this objective using an easy money policy of buying government securities.
This change in turn affected the Federal funds rate (the rate banks charge each other for
overnight loans) and increases excess reserves at banks. The fall in the Federal fund rate
in turn affects on the prime interest rate (the rate that banks charge their most credit-
worthy customers), and other interest rates. [text: E p. 296; MA pp. 296]

28. What is the relationship between the Federal funds rate and the prime interest rate?
Why doesn’t the Federal Reserve target the prime interest rate?

The prime interest rate is the rate that banks charge their most creditworthy customers.
It rises and falls with the Federal funds rate. When the Federal Reserve seeks to
increase or decrease the Federal funds rate, it winds up increasing or decreasing the
prime interest rate, and thus influencing the whole economy. The Federal funds rate is
easier for the Federal Reserve to influence immediately given its direct control over
bank reserves. The Federal funds rate is set by the market for excess bank reserves.
[text: E p. 296; MA pp. 296]

29. How is the Federal funds rate established? What role does the Federal Reserve play?

The Federal funds rate is established in the market for overnight excess reserves held by
banks. It is based on the supply and demand for excess reserves. The Federal funds rate
has been the recent target of monetary policy. The Federal Reserve can influence the
Federal funds rate by buying or selling government bonds. When the Federal Reserve
buys bonds, this action increases the supply of excess reserves of banks. The Federal
funds rate falls so it becomes cheaper for banks to borrow excess reserves overnight.
Conversely, when the Federal Reserve seeks to increase the Federal funds rate, it sells
bonds and this action reduces the excess reserves of banks. As a consequence, the
Federal funds rate rises so it becomes more expensive for banks to borrow excess
reserves overnight. [text: E p. 296; MA p. 296]

30. What is the predicted net export effect of a tight monetary policy? Explain.

A tight monetary policy will cause interest rates to rise and as a result the inflow of
foreign financial capital into the United States should increase. As foreign investors
demand dollars to invest in the U.S. this causes the dollar to appreciate relative to other
currencies. When the dollar appreciates, American goods become more expensive to
foreign buyers and foreign goods and services become less expensive to American
buyers. Therefore, exports tend to fall and imports tend to rise which reduces the value
of net exports. Exchange rate changes in response to interest rate changes in the United
States strengthen domestic monetary policy. [text: E pp. 297-298; MA pp. 297-298]

31. Explain how the net export effect should strengthen the effects of either an easy or a
tight monetary policy.

With an easy monetary policy interest rates should decline as the money supply
expands. As interest rates drop, fewer foreign investors will want American securities
because of the decline in the rate of return. This, in turn, reduces the demand for dollars
and causes a depreciation of the dollar. The cheaper dollar makes American goods more
attractive to foreigners and foreign goods more expensive to Americans. This tendency
for exports to grow and imports to decline results in a positive change in net exports
which expands aggregate expenditures further.

206
Technology, R&D, and Efficiency

A tight monetary policy causes just the opposite changes in the demand for American
securities. The increased demand will cause an appreciation of the dollar, American
goods become more expensive to foreigners and foreign goods become less expensive
for Americans. This tendency for exports to decline and imports to expand will reduce
net exports and cause a decline in aggregate expenditures which reinforces the effects of
the tight monetary policy. [text: E pp. 297-298; MA pp. 297-298]

32. Describe the links between monetary policy and the international economy in the net
export effect.

Net export effect occurs when foreign financial investors respond to a change in interest
rates. Higher interest rates lead to appreciation of dollar value in the foreign exchange
markets; lower interest rates from an easy monetary policy will lead to dollar
depreciation in the foreign exchange markets. When the dollar appreciates, U.S. goods
become more costly to foreigners, and this lowers demand for U.S. exports, that in turn
tends to reduce GDP. This result is the desired effect of a tight money policy.
Conversely, an easy money policy leads to depreciation of the dollar, greater demand for
U.S. exports and higher GDP.

Both easy and tight money policies have an effect on a trade deficit. An easy money
policy tends to increase net exports and thus it tends to correct a trade balance at the
same time as increasing GDP. The reverse is true for a tight monetary policy. It
reduces net exports because of the appreciated dollar and thus worsens a trade deficit. It
also reduces GDP. [text: E pp. 297-298; MA pp. 297-298]

33. (Last Word) What metaphors have been used by the popular press to describe the Fed
and its chairman?

The Fed has been depicted as a mechanic, with references to loosening or tightening
things, and to the economy running beautifully or acting sluggish, accelerating, or going
out of control. The warrior metaphor has been used—fighting inflation, plotting
strategy, and protecting the dollar from attack. The Fed has been depicted as the fall
guy in terms of administration officials “leaning heavily” on it and telling the Fed to
ease up or to relax. As a cosmic force, the Fed satisfies three criteria—power, mystery,
and a New York office. [text: E p. 299; MA p. 299]

CHAPTER 16 Extending the Analysis of Aggregate Supply

1. What is the basic difference between the short run and long run as these terms relate to
macroeconomics? Why does this difference occur?

The short run is a period in which nominal wages (and other input prices) remain fixed
as the price level changes. The long-run is a period in which nominal wages are fully
responsive to changes in the price level. Nominal wages tend to remain fixed in the
short run as the price level increases because workers may not be fully aware of how
inflation has eroded real wages. Also many workers are under fixed contracts for
several year periods. As a consequence of these factors, nominal wages do not change
immediately with changes in the price level. [text: E pp. 306-307; MA pp. 306-307]

4. Assume that one year the nominal wage for a worker is $12 per hour and there is no
inflation. The next year the nominal wage stays the same but the rate of inflation is 10
percent. What is the new real wage after taking inflation into account? What nominal
wage would workers ask for to keep their real wage equal to what it was the first year?

207
Technology, R&D, and Efficiency

The first year the nominal wage and the real wage are the same: $12 per hour. The
second year the nominal wage is $12, but the real wage is now $10.91. (To obtain this
number you take the $12 nominal wage and divided it by 1.10, which is the 10 percent
inflation expressed as price index in hundredths). To compensate for inflation, workers
would want the nominal wage of $12 to increase by 10 percent, or rise to $13.20 [$12 +
($12 x 1.10) = $13.20]. This increase in the nominal wage to $13.20 would make the
new real wage ($13.20/1.10 = $12) equal to the original $12 real wage. [text: E p. 307;
MA p. 307]

6. Describe the characteristics of the short-run aggregate supply curve. Explain what
happens to: (1) nominal wages; (2) real wages; (3) employment; (4) output; (5)
revenues; and, (6) profits as the price level increases from the full-employment level of
output. Then explain what happens to these variables as the price level decreases from
the full-employment-level of output.

The short-run aggregate supply curve will be an upsloping curve with the price level on
the vertical axis and real domestic output on the horizontal axis. The initial level of
output will be the full-employment level of output.

As the price level increases from the full-employment level of output, revenues to the
firm increase because nominal wages are fixed, and the profits for firms will rise. Firms
will have an incentive to increase output and employment (hiring temporary or part-time
workers or paying for overtime), so real GDP will increase and unemployment will fall
below its natural rate.

As the price level decreases from the full-employment level of output, revenues to the
firm decrease and because nominal wages are fixed, the profits for firms will decrease.
Firms will have an incentive to decrease output and employment, so real GDP will
decrease and employment will fall below its natural rate. [text: E pp. 307-308; MA pp.
307-308]

9. Describe the characteristics of the long-run aggregate supply curve. Explain how
changes in the price level affect the short-run aggregate supply curve and the long-run
aggregate supply curve.

The long-run aggregate supply curve will be vertical at the full-employment level of
output. Changes in the price level will not affect the full-employment level of output in
the long-run. Changes in the short-run aggregate supply curve will define the long-run
aggregate supply curve at the full-employment level of output.

With the short-run aggregate supply curve, as the price level increases from the full-
employment level of output along the curve, revenues to the firm increase because
nominal wages are fixed, and the profits for firms will rise. Firms will have an incentive
to increase output and employment (hiring temporary or part-time workers or paying for
overtime), so real GDP will increase and unemployment will fall below its natural rate.
This situation is a short-run one because nominal wages (and other input prices) will
eventually increase and shift the short-run aggregate supply curve to the left. The new
equilibrium will return to the full-employment level of output, but at a higher price
level.

Conversely, as the price level decreases from the full-employment level of output,
revenues to the firm decrease and because nominal wages are fixed, the profits for firms
will decrease. Firms will have an incentive to decrease output and employment, so real
GDP will decrease and employment will fall below its natural rate. This situation is a
short-run one because nominal wages (and other input prices) will eventually decrease
and shift the short-run aggregate supply curve to the right. The new equilibrium will

208
Technology, R&D, and Efficiency

return to the full-employment level of output, but at a lower price level. [text: E pp.
307-309; MA pp. 307-309]

10. What is the long-run equilibrium in the extended aggregate demand and aggregate
supply model?

The equilibrium GDP and price level occur at the intersection of the aggregate demand
curve, the long-run aggregate supply curve, and the short-run aggregate supply curve.
The output level will be at the full-employment level of output. [text: E pp. 308-309;
MA pp. 308-309]

11. Describe the process that occurs with demand-pull inflation in the extended aggregate
demand and aggregate supply model.

Assume that the economy is initially in equilibrium at the full-employment level of real
output. If the price level rises because of an increase in aggregate demand, then this
event will cause a movement along the short-run aggregate supply curve. The revenues
and profits of firms increase because nominal wages and the prices of other resources
are fixed. Employment and output will increase beyond the full-employment level to a
temporary equilibrium. In the long-run, once workers and resource suppliers realize that
the price level has risen they will want higher prices for their resources. When these
higher payments are made, the short-run aggregate supply curve will shift to the left.
This change will eventually result in a new equilibrium at a higher price level with real
output and employment returning to its full-employment level. [text: E pp. 309-310;
MA pp. 309-310]

13. Differentiate between “demand-pull” and “cost-push” inflation in the basic aggregate
demand and aggregate supply model.

Demand-pull inflation occurs when an increase in aggregate demand pulls up the price
level. Graphically, the demand curve is shifting rightward in the intermediate or
classical range of the aggregate supply curve. Cost-push inflation is a result of
aggregate supply decreasing relative to aggregate demand. Graphically, the aggregate
supply curve would be shifting leftward, intersecting the aggregate demand curve at a
higher level of prices. [text: E pp. 309-311; MA pp. 309-311]

209
Technology, R&D, and Efficiency

14. Explain what happens in the extended aggregate demand and aggregate supply model
when there is a recession.

With a recession, the aggregate demand curve will decrease or shift left. As the price
level decreases from the full-employment level of output, revenues to the firm decrease,
and because nominal wages are fixed, the profits for firms will decrease. Firms will
have an incentive to decrease output and employment (to reduce labor cost), so real
GDP will decrease and employment will fall below its natural rate. This situation is a
short-run one, however, because nominal wages (and other input prices) will eventually
decrease and shift the short-run aggregate supply curve to the right. The new
equilibrium will return to the full-employment level of output but be at a lower price
level. [text: E p. 311; MA p. 311]

16. What is the Phillips Curve? What concept does it illustrate?

The Phillips Curve shows the relationship between the unemployment rate and the rate
of inflation. The relationship is an inverse one, so there is a short-run tradeoff between
the unemployment rate and the rate of inflation. For example, if the unemployment rate
increases by 1% then the inflation rate might decline by .5%. The concept was
developed by A.W. Phillips in Great Britain based on empirical observation of the
relationship between unemployment and inflation in that nation. Modern economists
reject the idea of a stable, predictable Phillips Curve, although many economists do
agree that there is a short-run tradeoff between unemployment and inflation. [text: E pp.
312-313; MA pp. 312-313]

18. If the Phillips Curve exists in reality, what dilemma does this create for fiscal and
monetary policies? Explain.

The dilemma is that an expansionary fiscal and monetary policy aimed at reducing
unemployment may cause inflation, and vice versa for policies aimed at reducing
inflation. If there truly is a tradeoff, then successful elimination of unemployment
cannot be accomplished without creation of inflation; likewise, stable prices occur only
in the presence of some unemployment. [text E p. 312; MA p. 312]

19. What is stagflation and what was one of its causes in the 1970s and early 1980s?

Stagflation is the presence of both inflation and unemployment over a period of time
such as occurred in the 1972–1974 and 1977–1980 periods. The major cause appears to
be a series of adverse aggregate supply shocks that shifted the economy's short-run
aggregate supply curve to the left. One of the major shocks was the quadrupling of oil
prices by the Organization of Petroleum Exporting Countries (OPEC) that significantly
increased energy prices. Other shocks included severe agricultural shortfalls around the
world, the depreciation of the dollar, wage and price increases after wage-price controls
were removed, and a decline in productivity. [text E pp. 313-314; MA pp. 313-314]

21. What contributed to stagflation's demise between 1982 and 1989? How did these events
affect aggregate supply and the Phillips Curve?

There was a deep recession from 1981 to 1982 that reduced the pressure for workers to
increase wages and for firms to increase prices. Increased foreign competition also
restrained domestic wage and price increases in the United States. The U.S. economy
also underwent deregulation in many basic industries that stimulated more competition
and further restrained price and wage increases. In addition, the monopoly and pricing
power over oil by the OPEC cartel was weakened, thus reducing energy prices for
consumers and businesses. These factors worked to reduce the per-unit cost of

210
Technology, R&D, and Efficiency

production and shift the economy's short-run aggregate supply curve to the right and
shift the Phillips Curve back to the left. [text E p. 314; MA p. 314]

211
Technology, R&D, and Efficiency

24. Why is the difference between the actual and expected rates of inflation important for
explaining inflation?

When the actual rate of inflation is higher than the expected rate of inflation, profits
temporarily rise because prices that firms charge for their products are rising faster than
wage rates. (The nominal wage rates were based on a lower expected rate of inflation
than actually exists.) With more revenues, firms can afford to employ more workers so
the unemployment rate temporarily falls. In the long run, firms and workers adjust their
expectations to the new higher rate of inflation. This means that there will be an
increase in the nominal wages rate, so the profits decrease. The firm cannot afford to
hire as many workers so some workers get laid off. The unemployment rate rises and
returns to its natural rate. [text: E pp. 315-316; MA pp. 315-316]

27. Explain the basic arguments for supply-side economics.

The basic tenet of supply-side economics is that macroeconomic policies have focused
on aggregate demand while ignoring the impact of those policies on aggregate supply.
Supply-siders contend that changes in aggregate supply are just as active a force in
determining employment and price levels as are changes in aggregate demand. In
particular, they argue that high marginal tax rates, public transfer programs, and
overregulation have caused the aggregate-supply curve to shift leftward while the
aggregate-demand curve is being shifted rightward. [text: E p. 317; MA p. 317]

28. What is the Laffer Curve? Explain the relationship that is shown in the curve.

The Laffer Curve indicates that the relationship between tax rates and tax revenues is
not a clear one. At the extreme, both a zero rate and 100% tax rate will produce zero
revenue. In between the two extremes there is an optimal tax rate in terms of
maximizing revenue. If tax rates are above the optimal level, then tax revenues will rise
as tax rates are cut. If tax rates are below the optimal level then tax revenues will fall as
tax rates are cut. Laffer argued that tax rates were above the optimal level and that by
lowering tax rates, the government could increase tax revenue and increase economic
output at the same time. [text: E pp. 317-318; MA pp. 317-318]

CHAPTER 17 Economic Growth and the New Economy

1. Identify the two major ways economic growth is measured.

Economic growth can first be defined and measured as an increase in real GDP
occurring over a period of time. Economic growth can also be defined and measured as
an increase in real GDP per capita occurring over a period of time. Economic growth is
usually calculated as an annual percentage rate of growth. [text: E pp. 136, 323; MA pp.
136, 323]

2. Summarize the four supply factors in economic growth.

The four supply factors in economic growth are the quantity and quality of the natural
resource base, the quantity and quality of the labor force, the supply or stock of capital
goods, and the state of technology. Expansion or improvement in any of these areas will
contribute to more rapid economic growth. [text: E p. 323; MA p. 323

3. Explain the demand factor in economic growth.

Aggregate demand must be growing and sufficient to purchase the expanding output of
the economy. When this occurs, there will be no unplanned increases in inventories and

212
Technology, R&D, and Efficiency

resources will be fully employed. Total spending in the economy needs to match total
production. [text: E pp. 323-324; MA pp. 323-324]

4. What is the efficiency factor?

This factor means resources are used in the least costly way (productive efficiency) and
are used to produce the mix of products most wanted by society (allocative efficiency).
[text: E p. 324; MA p. 324]

6. The achievement of full employment is a sufficient condition for the achievement of


economic growth. Evaluate.

Full employment is not a sufficient condition for economic growth. Underdeveloped,


no-growth economies may have full employment in the sense that everyone is working
at providing a subsistence level of living, but this does not lead to economic growth.
Also important here is the allocative factor. Not only is full employment of resources
required, but resources must also be employed efficiently so they produce maximum
levels of goods and services. [text: E pp. 323-324; MA pp. 323-324]

8. Define worker-hours and labor productivity. What factors are behind labor
productivity?

Worker-hours is the measure of work defined as the number of workers multiplied by


the amount of time they work in a particular time period for example, a year. Labor
productivity is real output per worker-hour; that is, the value of real output divided by
the number of worker-hours.

Productivity is determined by technological progress, the amount and quality of capital


goods per worker, the quality of labor itself, and the efficiency with which inputs are
allocated, combined, and managed. [text: E p. 325; MA p. 325]

9. Assume that an economy has 2,000 workers, each working 3,000 hours per year. The
average real output per worker-hour is $10. What will the total output or real GDP be?
Explain.

$60 million. Take the 2,000 workers and multiply this times 3,000 work hours. This
results in 6,000,000 work hours. Multiply this number of work hours by $10 per hour
results in $60 million. [text: E p. 325; MA p. 325]

14. In the diagram below, the economy is initially at Q1 and after a period of economic
growth it will produce at Q2. Explain the path of economic growth including impact on
output and prices as indicated by this diagram.

213
Technology, R&D, and Efficiency

Both long-run and short-run aggregate supply increased over time. Also, aggregate
demand increased. The combined effects of these shift has been an increase in output
from Q1 to Q2 and an increase in the price level from P1 to P2. [text: E pp. 326-327; MA
pp. 326-327]

15. Summarize the historical growth record of the United States over the past 50 years in
terms of growth in real GDP and in real per capita GDP.

The real GDP has increased about 3.5% per year. Real per capita GDP has risen more
slowly because population has grown along with GDP. Still real GDP per capita has
expanded at approximately 2.3% per year. [text: E p. 327; MA p. 327]

16. Which is more important—labor quantity or labor productivity—as a source of


economic growth in the United States? Explain.

Both labor quantity and labor productivity contribute significantly to increased


economic growth in the United States. About a third of the increase in economic growth
comes from labor quantity and about two-thirds from labor productivity. Clearly, it is
not the quantity of labor that is most important but the quality of that labor and how it is
used for production. [text: E p. 328; MA p. 328]

18. What is meant by economies of scale and what is the importance of this concept to
economic growth?

Economies of scale are production advantages which are derived from market and firm
size. The importance of this concept is that large manufacturers are able to use
assembly-line production techniques, to use large-scale efficient equipment, and to take
advantage of specialization that cannot be done in small-scale production. This
efficiency in production and distribution contributes to growth. [text: E pp. 330-331;
MA pp. 330-331]

19. How are real output, real income, and real wages linked to labor productivity?

Increasing labor productivity means that the standard of living can improve. Greater
output per worker means there are more goods and services produced in an economy.
When there are more goods and service produced, there are more goods and services
that can be sold by businesses, which in turn receive higher revenues. Businesses can
then afford to pay workers higher wages because they contribute more production to
business firms. Labor productivity thus determines the average real wage in an
economy. Real wages are also a key determinant of real income in an economy. [text:
E p. 331; MA p. 331]

22. Use productivity to explain why some economists think a “new economy” has been
established in the United States.

Productivity growth was 3.1 percent annually from 1995–2000 compared with 1.4
percent annually from 1973–1995. This development caused some economists to think
that a “new economy” had emerged based on improvements in technology and global
competition. These economists think that it might be possible for the economy to grow
at a 3 to 4 percent annual rate without igniting inflation. Improvements in productivity
permit faster economic growth. [text: E p. 331; MA p. 331]

23. What are the characteristics of the New Economy that distinguish it from the economy
of previous periods?

214
Technology, R&D, and Efficiency

First, there has been an expansion of entrepreneurship and innovation based on the
information technologies arising from the microchip and other information resources
such as the Internet and communication devices. Second, there are many new startup
firms that were not in business 25 years ago that are taking advantage of the new
information technology. These businesses often experience increasing returns to scale.
Third, global competition has reawakened capitalism around the world. Trade barriers
have fallen and former socialist nations are now making a transition to market-based
economies. [text: E p. 333-334; MA pp. 333-334]

25. Does the New Economy mean that the business cycle is dead?

No. The basic idea is that the trend line of economic growth is steeper for the New
Economy. Nevertheless, it can still experience recession, which means that real output
can fall below the now steeper trend line for a period of time. [text: E p. 334; MA p.
334]

CHAPTER 18 Deficits, Surpluses, and the Public Debt

1. Differentiate between the Federal deficit and the Federal debt.

The Federal deficit is an annual concept referring to the shortfall between Federal
revenues and expenditures in one year’s budget. The Federal debt is the accumulation
of borrowing which results from the series of deficits minus any surpluses. [text: E p.
341; MA p. 341]

2. The following table shows government spending and tax revenue for a hypothetical
economy over a five-year period. All figures are in billions.

Year Government Spending Tax Revenues


1 $ 800 $825
2 850 850
3 900 875
4 950 900
5 1,000 925

(a) In what years were there budget deficits and what were the amounts?
(b) In what year was there a budget surplus and what was the amount?
(c) What is the public debt in this economy over the five years?

(a) There were budget deficits in year 3 (–$25 billion), year 4 ($50 billion) and year 5
($75) billion.
(b) There was budget surplus in year 1 ($25 billion).
(c) The public debt was $125 billion. (+$25 billion +$0 –$25 billion –$50 billion –$75
billion) [text: E p. 341; MA p. 341]

3. Compare and contrast the three budget philosophies: annually balanced budget,
cyclically balanced budget, and functional finance.

The annually balanced budget philosophy advocates that government revenues should
equal expenditures on an annual basis. The cyclically balanced budget philosophy is
similar in that it advocates a balanced budget, but over a longer period of time, allowing
for deficits during the recessionary phases and surpluses in the prosperous, peak phases
which would compensate for the deficits. Functional finance ignores the relationship
between taxes and spending in the Federal budget, preferring to focus on the state of the
economy and spending and tax policies which are appropriate to stabilize it.

215
Technology, R&D, and Efficiency

With regard to the use of fiscal policy as a stabilization tool, both the functional finance
and cyclically balanced budget philosophies are similar in their belief that fiscal policy
can be useful in stabilizing employment and prices. The annually balanced budget
philosophy believes that the growth of government should and would be limited by this
policy. Mainstream economists, however, point out that an annually balanced budget
would be procyclical and intensify the effects of the business cycle. [text: E pp. 341-
342; MA pp. 341-342]

4. What is the problem with a cyclically balanced budget?

Balancing the budget over the business cycle seems like a good idea in principle, but it
is difficult to achieve in practice. The ups and downs of the business cycle may not be
of equal magnitude and duration. The economy could be in a slump for a long time
period and then enjoy prosperity for just a few years. The accumulated budget deficit
during the slump will be much greater than any accumulated budget surpluses during a
short expansion. This situation would leave a perpetual cyclical deficit in the budget.
[text: E p. 342; MA p. 342]

New 5. Why is an annually balanced budget not compatible with government fiscal activity
viewed as a countercyclical, stabilizing force in the economy?

The annually balanced budget philosophy rules out using fiscal policy as a
countercyclical, stabilizing force. During a recession, the government would have to
raise taxes and lower spending to balance the budget as tax revenues fell because people
had lower incomes. During an inflationary boom, a balanced budget would intensify the
inflation. As tax revenues increased, the government would need to cut taxes or
increase spending to avoid a budget surplus. This strategy would make the inflation
worse. The balanced budget is not neutral, but is procyclical, which means it intensifies
the direction of the business cycle. [text: E p. 342; MA p. 342]

New 6. What is the cyclically balanced budget? What is its major problem?

The cyclically balanced budget would balance the government budget over the course of
the business cycle. This philosophy would give the government the option to pursue a
countercyclical stabilization policy during the course of the business cycle. Deficit
spending would be used to counter the effects of recession, and surpluses would be used
to restrain inflation. The major problem with this philosophy is that the rises and falls of
the business cycle are not of equal magnitude or duration, so it is difficult to balance the
government budget over time. [text: E p. 342; MA p. 342]

New 7. Briefly explain functional finance.

Functional finance suggests that the condition of the budget is secondary to the
condition of the economy. In this view, the primary purpose of Federal finance is to
achieve economic growth, full employment, and price level stability. Government
should do what is necessary to achieve those goals regardless of whether such actions
produce deficits or surpluses in the budget. [text: E p. 342; MA p. 342]

8. State three causes of the public debt.

National defense and military spending have soared over the years since World War II.
Recessions cause a decline in revenues and growth in government spending which have
not been reversed in prosperous periods. The tax cuts of the 1980s without equivalent
spending cuts have increased the deficit because they have not led to revenues rising as
much as had been predicted. [text: E pp. 343-344; MA pp. 343-344]

216
Technology, R&D, and Efficiency

9. What information would be important for assessing the size of the public debt beside the
absolute amount of the public debt?

When the public debt is stated as an absolute amount, the numbers can often be
staggering and misleading. The absolute amount, however, needs to be viewed in
relation to the size of the economy (its GDP). If GDP is large relative to the public debt,
then the nation has the capacity to service that debt. Also, as the economy grows, the
debt as a percentage of GDP will tend to fall unless the economy continues to add
substantially to the public debt each year with large budget deficits. [text: E p. 344; MA
p. 344]

11. What is the relationship between social security and the public debt?

Social security currently generates more revenues than it pays out to retirees. The
excess funds are used to purchase government securities and hold them in a social
security trust fund. The excess funds also are included in the calculation of the
government budget. Thus, they have the effect of reducing the size of any budget deficit
or increasing the size of a budget surplus. Some economists argue that these funds
should be excluded from calculating annual deficits or surpluses because the excess
social security funds are accumulating to pay for the future spending needs for the
program. [text: E p. 345; MA p. 345]

12. Can the large public debt cause the nation to go bankrupt? Explain.

The large Federal debt cannot cause the country to go bankrupt. For one thing the size
of the debt relative to the nation’s earning power as measured by GDP is not higher
today than it has been in the past. The government can refinance bonds when they
mature by selling new bonds in the same way that corporations often do. The
government has the power to tax which could raise more revenue if necessary. [text: E
pp. 345-346; MA pp. 345-346]

13. Adam Smith once wrote: “What is prudence in the conduct of every private family can
scarce be folly in that of a great kingdom.” Evaluate in terms of the national debt.

This is an example of the fallacy of composition. What is true for the individual is not
necessarily true for governments when it comes to borrowing. Government can
refinance its debt. The government, unlike individuals, also has the power to tax. [text:
E pp. 345-346; MA pp. 345-346]

15. Why is the ownership of the public debt an important issue?

There are two important possibilities here: (1) If the debt is held primarily by the
wealthy in our society, then the interest payment on the debt contributes to income
inequality as taxpayers pay the interest to those wealthy bondholders; and (2) If the debt
is held substantially by those outside the country, then the interest payment on the debt
represents a drain of resources from this country to others and there is a dependence on
these lenders that may restrict U.S. independence. [text: E pp. 346-347; MA pp. 346-
347]

18. What are four real and potential problems with the public debt?

Four real and potential problems with public debt are mentioned in the text: (a)
Repayment of the debt affects income distribution in that working taxpayers will be
paying interest to those who hold the debt. If those who hold the debt are primarily in
the upper-income groups, income distribution may be made more unequal; (b) If the tax

217
Technology, R&D, and Efficiency

burden of paying the interest on the debt becomes excessive, it may damage incentives
to work, save, and invest; (c) Debt owed to foreign investors could increase the nation’s
burden since interest payments leave the country. It may also make our financing less
stable if dependence on foreign lending increases; (d) Some public borrowing may
“crowd out” private borrowing. [text: E pp. 346-348; MA pp. 346-348]

19. If the public debt is a debt that we owe to ourselves, then there are obviously no
problems connected with such a debt. Critically evaluate.

While the fact that we owe much of the debt to ourselves may alleviate much concern
about the size of the public debt, it does not mean that there are no problems connected
with it. Several difficulties exist: the owners of the debt tend to be those in the higher
income categories and payment of interest on the debt then contributes to income
inequality; if the ratio of interest payments to GDP grows, the nation’s tax burden rises
which may have some impact on work incentives; financing the debt requires public
borrowing which could crowd out private investment borrowing and spending.

Looking at some of the above-mentioned problems, the public debt could be inflationary
if the expansionary policies which cause it to grow also cause aggregate demand to
expand more rapidly than aggregate supply. The public debt could be inflationary if it
has a negative supply-side impact through restraining private capital investment both of
which would tend to reduce productivity and raise production costs. [text: E pp. 346-
348; MA pp. 346-348]

22. What two factors could reduce the net economic burden that might be shifted to future
generations from the public debt?

If the debt borrowing is for public investments, such as infrastructure or in human


capital, that in turn causes the economy to be more productive in the future, then the
burden on future generations will be less compared to the situation where the
government had not borrowed for this public investment purpose. Also, if the public
borrowing is used for projects that in turn encourage private investment, then there are
complementarities that will reduce the burden on future generations because they will
inherit a larger capital stock. [text: E pp. 347-348; MA pp. 347-348]

23. Changes in the size of the public debt have a much greater impact on the economy than
does the mere existence of a large debt. Evaluate.

Certainly changes in the size of the public debt could have a greater impact on the
economy under some circumstances. Such changes alter AD, whereas the existence of
the debt does not. But this would not always be the case.

The existence of the public debt does not change aggregate demand and therefore does
not add to economic instability. In contrast, changes in the size of the public debt imply
current budget deficits or surpluses which alter aggregate demand. Changes in
aggregate demand affect real output, the price level, or both. Therefore, the statement is
true. [text: E pp. 348-349; MA pp. 348-349]

24. Describe the actions taken by government to reduce the budget deficits of the 1990s.

In 1993, the Clinton administration passed the Deficit Reduction Act that increased tax
revenues by $250 billion over five years and sought to reduce federal spending by the
same amount. The legislation increased marginal tax rates from 31 percent to 39.6
percent for individuals; raised corporate income tax rates from 34 to 35 percent; put an
excise tax on gasoline; and sought to hold discretionary spending to 1993 levels.

218
Technology, R&D, and Efficiency

Further fiscal restraint by Congress helped achieve a budget surplus in 1998, the first
time there had been one since 1969. [text: E p. 349; MA p. 349]

219
Technology, R&D, and Efficiency

25. Discuss the pros and cons of paying down the public debt with the projected surpluses.

The supporters contend that paying down the public debt will contribute to economic
growth. The actions will create a reverse crowding out effect in which there will be
lower real interest rates that will stimulate more investment and produce a greater
capital stock for the nation. The policy might also help reduce the nation’s trade deficit.
Critics counter that paying down the debt may not be all that useful. The critical
objective is to have the debt decline as a percentage of GDP. As the economy grows,
this decline will occur on its own and create sounder fiscal conditions for government.
The critics also think that there may be less of a reverse crowding out effect than is
thought because of the complexity of the economy. Finally, they suggest that there is a
positive role for debt in the economy for managing social security and for conducting
monetary policy. [text: E p. 349; MA p. 349]

26. Evaluate the advantages and disadvantages of tax cuts for the disposing of budget
surpluses.

Advocates for tax cuts state that surpluses were the result of excess tax revenues that
should be returned to taxpayers. The other argument for tax cuts is to restrain
government spending. The idea is that if tax revenues are available, then the surplus
will be spent on new government programs. Tax cuts, in effect, put government on a
restricted diet. Another argument that is made for tax cuts is that they can stimulate the
economy during a period of slow growth or recession. Critics counter that a permanent
reduction in taxes is not wise when the projections for surpluses are tentative. If the
surpluses do not materialize as completely as projected, then tax cuts can create budget
deficits that add to the public debt. Also, cutting taxes when the economy is growing at
a fast rate can be inflationary. [text: E p. 350; MA p. 350]

CHAPTER 19 Disputes over Macro Theory and Policy

1. Compare and contrast the classical and Keynesian views of aggregate demand and
supply.

The classical view is that the aggregate supply curve is vertical and that it alone
determines the level of real output. The level of real output (and employment) does not
change as aggregate demand changes because there is perfect flexibility in wages and
prices. Thus it is not possible for the economy to remain at less than the full-
employment level of output for any length of time. Also, the aggregate demand curve is
downsloping and it alone determines the price level. The aggregate demand curve,
however, will be stable if monetary authorities maintain a constant money supply.

The Keynesian view is that the aggregate supply curve is horizontal up to the full
employment level of real output because there is downward inflexibility in prices and
wages. Therefore, a decline in real output will have no effect on the price level.
Keynesian economists also think that the aggregate demand curve is unstable and
subject to periodic fluctuations, in large part because of the instability of the investment
component of aggregate demand. If aggregate demand decreases in the horizontal range
of aggregate supply, then real output will decrease, but the price level remains constant.
In this case, it is possible for the economy to achieve equilibrium at less than the full-
employment level of output, a result that is not possible in the opinion of classical
economists. [text: E pp. 355-357; MA pp. 355-357]

220
Technology, R&D, and Efficiency

2. The following questions refer to graphs A and B below. In the graphs, Qf represents
full-employment output and Qu1 and Qu2 represent less-than-full-employment levels of
output.

(a) Which of the two graphs best illustrates the Keynesian view of the macroeconomy,
and which best illustrates the classical view? Explain.
(b) When demand shifts from AD1 to AD2, explain what happens to output and price
level in graph A.
(c) When demand shifts from AD1 to AD2, explain what happens to output and price
level in graph B.

(a) Graph A represents the classical view and Graph B represents the Keynesian view.
In Graph A full employment is the norm and any change in demand is reflected in
changing price levels. This is the classical perspective. However, in Graph B a change
in demand has an impact on real output and may or may not affect the price level.
(b) Output remains at full employment but price level rises.
(c) Output declines but price level remains the same. [text: E pp. 355-357; MA pp.
355-357]

3. Describe two basic differences between the Keynesian and monetarist economic
theories.

Keynesians believe that the capitalist economy is inherently unstable and that business
cycle fluctuations could lead to periodic inflation or unemployment. As a result they
support government intervention to assist in stabilizing the economy.

Monetarists believe that capitalist markets are highly competitive and that this
competition makes the economy very stable. Prices and wages fluctuate to equilibrate
the economy at a level of full employment. [text: E pp. 355-357; MA pp. 355-357]

5. Explain the equation of exchange.

The fundamental monetarist equation is MV = PQ. In this equation M is the money


supply and V is the velocity at which the average income dollar is spent in a year, MV
represents the total amount spent by purchasers. P is the price level and Q is the
physical volume of goods and services produced. PQ, therefore, is the value of nominal
GDP. [text: E pp. 359-360; MA pp. 359-360]

6. Assume that M is $500 billion and V is 5. What is the level of nominal GDP according
to the monetarist equation? If V rises by 10%, then according to the monetarist
equation, what will be the new level of nominal GDP?

221
Technology, R&D, and Efficiency

5 times $500 billion = $2500 billion GDP. If V rises by 10% to 5.5, then the new level
of GDP should rise by 10% to $2750 billion or 5.5 times $500 billion = $2750 billion.
[text: E pp. 359-360; MA pp. 359-360]

10. Assume that M is $250 billion and V is 8. If V increases by 15%, what will be the
change in nominal GDP?

The nominal GDP should rise by 15%. The arithmetic is as follows: Original GDP =
$2000 billion; after V rises to 9.2, GDP = $2300 billion. [text: E pp. 359-360; MA pp.
359-360]

11. If nominal GDP is $1344 billion, and velocity of money is 6, how much is the money
supply? If the GDP price index is 160, what is real GDP here?

$1344 billion/6 = $224 billion = PQ, where Q is real GDP. If P = 1.60, then real GDP =
$1344/1.60 = $840 billion. [text: E pp. 359-360; MA pp. 359-360]

12. Define the velocity of money. Explain the monetarist view with regard to the stability
of velocity.

The velocity of money is the number of times per year that the average dollar is spent.
It can be viewed as the rate at which money turns over in a year. The monetarists look
at the value of V over the long run and conclude that it is stable. By stable the
monetarist does not mean “constant.” Rather, the monetarist thinks that the factors
changing velocity over time occur gradually and are predictable. Any year-to-year
changes in velocity can be easily anticipated. [text: E pp. 358-359; MA pp. 358-359]

13. Does velocity change in response to changes in the money supply according to
monetarists?

No. People have a stable desire to hold money relative to other financial assets, real
assets, or for buying goods and services. How much money the public wants to hold
depends primarily on the level of nominal GDP. Monetarists reason that the money
supply is the causal force in determining nominal GDP. [text: E pp. 358-359; MA pp.
358-359]

14. What are the four different views of the causes of macroeconomic instability in the
economy?

First, the mainstream view is Keynesian based and holds that instability in the economy
arises from: (a) the volatility in investment spending that makes aggregate demand
unstable; and (b) occasional aggregate supply shocks which cause cost-push inflation
and recession. Second, monetarists focus on the money supply, think markets are highly
competitive, and that government intervention destabilizes the economy. Monetarists
see macroeconomic instability as a result of inappropriate monetary policy. An increase
in the money supply will increase aggregate demand, output, and the price level; it will
also reduce unemployment. Eventually, nominal wages rise to restore real wages, and
real output and the unemployment rate falls back to its natural level at long-run
aggregate supply. Third, real-business cycle theorists see macroeconomic instability as
being caused by real factors influencing aggregate supply instead of monetary factors
causing shifts in aggregate demand. Changes in technology and resources will affect
productivity, and thus the long-run growth rate of aggregate supply. Fourth, another
view of instability in the macro economy attributes the reasons to coordination failures.
These failures occur when people are not able to coordinate their actions to achieve an
optimal equilibrium. A self-fulfilling prophecy can lead to recession because if
households and firms expect it, they individually cut back on spending and employment.

222
Technology, R&D, and Efficiency

If, however, they were to act jointly, they could take actions to counter the recession
expectations to achieve an optimal equilibrium. [text: E pp. 359-361; MA pp. 359-361]

16. Explain the new classical view of self-correction in the economy.

The new classical view of economics, held by monetarists and rational expectation
economists, is that the economy may deviate from the full-employment level of output,
but it eventually returns to this output level because there are self-corrective
mechanisms in the economy. Graphically, if aggregate demand increases, it temporarily
raises real output and the price level. Then, nominal wages rise and productivity falls,
so short-run aggregate supply decreases, thus bringing the economy back to its long-run
output level. There is disagreement about the speed of adjustment. The monetarists
adopt the adaptive expectations view that there will be a slower, temporary change in
output, but in the long-run it will return to its natural level.

Other new classical economists adopt the rational expectations theory (RET) that there
will be a rapid adjustment with little or no change in output. RET is based on two
assumptions: people understand how the economy works so that they quickly anticipate
the effect on the economy of an economic event; all markets in the economy are so
competitive that equilibrium prices and quantities quickly adjust to changes in policy.
In RET, unanticipated price-level changes, called price-level surprises, cause short-run
changes in real output because it causes misperceptions about the economy among
workers and firms. In RET, fully anticipated price-level changes do not change real
output even in the short-run because workers and firms anticipate and counteract the
effects of the changes. [text: E pp. 361-364; MA pp. 361-364]

17. Compare and contrast the new classical and the mainstream view of self-correction in
the economy.

The new classical view of economics, held by monetarists and rational expectation
economists, is that the economy may deviate from the full-employment level of output,
but it eventually returns to this output level because there are self-corrective
mechanisms in the economy. Graphically, if aggregate demand increases, it temporarily
raises real output and the price level. Then, nominal wages rise and productivity falls,
so short-run aggregate supply decreases, thus bringing the economy back to its long-run
output level. There is disagreement about the speed of adjustment. The monetarists
adopt the adaptive expectations view that there will be a slower, temporary change in
output, but in the long-run it will return to its natural level.
Other new classical economists adopt the rational expectations theory (RET) that there
will be a rapid adjustment with little or no change in output. RET is based on two
assumptions: people understand how the economy works so that they quickly anticipate
the effect on the economy of an economic event; all markets in the economy are so
competitive that equilibrium prices and quantities quickly adjust to changes in policy.
In RET, unanticipated price-level changes, called price-level surprises, cause short-run
changes in real output because it causes misperceptions about the economy among
workers and firms. In RET, fully anticipated price-level changes do not change real
output even in the short-run because workers and firms anticipate and counteract the
effects of the changes.

The mainstream view of self-correction suggests that price and wages may be inflexible
downward in the economy. Graphically, a decrease in aggregate demand will decrease
real output, but not the price level because nominal wages will not decline and cause the
short-run aggregate supply curve to shift right. The economy can get stuck in a
recession for a long time period.

223
Technology, R&D, and Efficiency

Downward wage inflexibility primarily arises because of wage contracts and the legal
minimum wage, but they may also occur from efficiency wages and insider-outsider
relationships according to mainstream economics. An efficiency wage minimizes the
firm’s labor cost per unit of output, but may be higher than the market wage. This
higher wage may result in greater efficiency because it stimulates greater work effort,
requires less supervision costs, and reduces job turnover. Insider-outsider relationships
may also produce downward wage inflexibility. During a recession, outsiders (who are
less essential to the firm) may try to bid down wages to try to keep their jobs, but the
firm may not lower wages because it does not want to alienate insiders (who are more
essential to the firm) and disrupt the cooperative environment in the firm that is needed
for production. [text: E pp. 361-365; MA pp. 361-365]

18. Describe the mainstream view of self-correction in the economy.

The mainstream view of self-correction suggests that price and wages may be inflexible
downward in the economy. Graphically, a decrease in aggregate demand will decrease
real output, but not the price level because nominal wages will not decline and cause the
short-run aggregate supply curve to shift right. The economy can get stuck in a
recession for a long time period.

Downward wage inflexibility primarily arises because of wage contracts and the legal
minimum wage, but they may also occur from efficiency wages and insider-outsider
relationships according to new Keynesian economics. An efficiency wage minimizes
the firm’s labor cost per unit of output, but may be higher than the market wage. This
higher wage may result in greater efficiency because it stimulates greater work effort,
requires less supervision costs, and reduces job turnover. Insider-outsider relationships
may also produce downward wage inflexibility. During a recession, outsiders (who are
less essential to the firm) may try to bid down wages to try to keep their jobs, but the
firm may not lower wages because it does not want to alienate insiders (who are more
essential to the firm) and disrupt the cooperative environment in the firm that is needed
for production. [text: E pp. 364-365; MA pp. 364-365]

19. How can paying workers an above-market wage result in greater efficiency? What are
the implications for the flexibility of wages?

An above-market wage raises the opportunity cost of losing a job. Workers thus have
more incentive to work hard and put more effort into their jobs. Higher wages also
reduce shirking on the job, so the firm does not have to hire as many supervisory
personnel to make sure workers are doing their jobs. The higher wage also reduces
turnover which can be costly to firms. The implication for the flexibility of wages is
that they would tend to be more inflexible if efficiency wages are paid. Employers
would be reluctant to cut wages because it can lower morale, increase shirking, and
increase turnover. [text: E pp 364-365; MA pp. 364-365]

23. What reasons do monetarists give for downgrading the importance of fiscal policy
relative to monetary policy?

Monetarists downgrade fiscal policy’s importance because of the so-called crowding-out


effect or because of the way in which fiscal policy is financed. If government finances
increased spending by running a deficit and selling bonds, then this government
borrowing will increase the demand for money, raise the interest rate, and crowd out
much private investment. Hence, the net effect of a budget deficit based on borrowing
from the public is weak. If the government finances the budget deficit through the
creation of new money, then there would not be crowding out, but the increase in the
money supply is really monetary policy, not fiscal policy in this case. [text: E pp. 365-
367; MA pp. 365-367]

224
Technology, R&D, and Efficiency

27. What rationale does rational expectations theory provide for the ineffectiveness of
discretionary policies?

Adherents contend that the aggregate responses of the public to its expectations will
render ineffective anticipated discretionary policies. For example, if monetary
authorities announce an easy money policy to increase output and employment, the
public will expect inflation in the future. Therefore, they will demand higher nominal
wages, businesses will increase product prices, and lenders will raise interest rates.
These actions will frustrate the attempt of policy makers to expand real output. The
only time that policy may be effective is when it is unanticipated, but this is virtually
impossible in a society with democratic institutions and free flow of information. [text:
E pp. 365-367; MA pp. 365-367]

28. Explain the mainstream economists’ justification for the use of discretionary fiscal and
monetary policy and their criticisms of policy rules.

Mainstream economists think that discretionary fiscal and monetary policy can be
effective and are opposed to a monetary rule and a balanced budget requirement. They
see velocity as relatively unstable and a loose link between changes in the money supply
and aggregate demand. This means that a monetary rule might produce too great a shift
in aggregate demand (and demand-pull inflation) or too small a shift (and deflation) to
match the shift in aggregate supply. Such a rule would contribute to price instability,
not price stability. They support the use of fiscal policy during a recession or to counter
growing inflation. Fiscal policy, however, should be reserved for those situations where
monetary policy is relatively ineffective. They also oppose a balanced budget
amendment because its effects would be procyclical and reinforce recessionary or
inflationary tendencies, rather than being countercyclical. Mainstream economists also
note that there has been greater stability in the macro economy since 1946 when
discretionary monetary and fiscal policy was being more actively used to moderate the
effects of the business cycle. [text: E pp. 367-368; MA pp. 367-368]
29. Identify how ideas from monetarism and rational expectations have been incorporated
into mainstream thinking about macroeconomics.

First, monetarists have gotten mainstream economists to recognize that changes in the
money supply are an important element in explaining long-lasting and rapid inflation.
Second, mainstream economists now recognize that expectations matter because of
rational expectation theory and because of theories coordination failures in the economy.
[text: E p. 368; MA p. 368]

30. Which aspects, if any, of monetarist or rational expectations theory have been integrated
into mainstream macroeconomics?

Mainstream economists today largely accept the monetarist position that the money
supply is an important variable affecting economic activity and that excessive growth
over a long period of time can be inflationary. The notion of “crowding out” of private
investment has been recognized as a weakness of expansionary fiscal policy without
coordinated monetary policy. Finally, policy makers have become much more sensitive
to expectations and the interaction of policy with particular markets in the
macroeconomy. [text: E p. 368; MA p. 368]

CHAPTER 20 International Trade

2. What are the major imports and exports of the United States?

225
Technology, R&D, and Efficiency

The principal exports of the United States are computers, semiconductors, chemicals,
consumer durables, and generating equipment, while its major imports are automobiles,
petroleum, computers, and clothing. [text: E p. 731; MA p. 375; MI p. 473]

3. Who are the major players in international trade?

The major participants in international trade are the United States, Japan, and the nations
of western Europe. Newer participants include Hong Kong, Singapore, South Korea,
Taiwan, and China. The collapse of the former Soviet Union changed trade patterns for
Russia and the nations of eastern Europe. Mainland China has also increased its trade.
[text: E p. 731; MA p. 375; MI p. 473]

4. Cite three important reasons why nations trade.

First, specialization and trade among nations is advantageous because the world’s
resources are not evenly distributed. To obtain resources or products that a nation does
not have but that are desired by society requires trade with other nations. Second, the
efficient production of different commodities necessitates different methods and
combinations of resources. These different methods and combinations can be obtained
through trade. Third, products vary in quality and in other ways. People prefer this
variety, which often can only be obtained through imports. [text: E pp. 731-732; MA
pp. 375-376; MI pp. 473-474]

7. Answer the next three questions on the basis of the following production possibilities
data for Francia and Galacia. All data are in tons.

Francia production possibilities:


A B C D E
Soup 60 45 30 15 0
Nuts 0 15 30 45 60

Galacia production possibilities:


A B C D E
Soup 20 15 10 5 0
Nuts 0 15 30 45 60

(a) If trade occurs between Francia and Galacia, which nation should export what
product? Why?
(b) What are the limits of the terms of trade between Francia and Galacia?
(c) Assume that prior to specialization and trade, Francia and Galacia chose production
possibility “C.” Now each specializes according to comparative advantage. What will
be the resulting gains from trade? Explain your answer.

(a) Francia should export soup and Galacia should export nuts. Francia is the low cost
producer of soup. The opportunity cost of 1 unit of soup is 1 unit of nuts. For Galacia
the opportunity cost of 1 unit of soup is 3 units of nuts.
Galacia is the low cost producer of nuts. The opportunity cost for Galacia to produce
nuts is 1/3 unit of soup. The opportunity cost of nuts for Francia is 1 unit of nuts for 1
unit of soup.
(b) From 1 unit of soup for 1 unit of nuts up to 1 unit of soup for 3 units of nuts.
(c) Before specialization, Francia produced 30 units and Galacia 10 units of soup for a
total of 40 units. After specialization and trade, 60 units of soup are produced by
Francia for a gain of 20 units of output.

226
Technology, R&D, and Efficiency

Before specialization, Francia produced 30 units and Galacia 30 units of nuts. After
specialization and trade, Galacia will produce 60 units. Output did not change with this
product. [text: E pp. 733-736; MA pp. 377-380; MI pp. 475-478]

11. State at least one economic benefit to increased international trade.

Benefits include increased specialization and efficiency, increased competition that also
should increase efficiency, provision of jobs in exporting industries, improved quality,
and lower prices for consumers where imports compete with domestic products. [text: E
p. 737; MA p. 381; MI pp. 479]

12. How can supply and demand analysis be used to explain the equilibrium price and
quantity of exports and imports for aluminum when there is trade between two nations
(e.g., the United States and Canada)?

For the United States, there will be domestic supply and demand and export supply and
import demand for aluminum. The price and quantity of aluminum are determined by
the intersection of the domestic demand and supply curves in a world without trade. In a
world with trade, the export supply curve for the United States shows the amount of
aluminum that American producers will export at each world price above the domestic
equilibrium price. American exports will increase when the world price rises relative to
the domestic price. The import demand curve for the United States shows the amount of
aluminum that Americans will import at each world price below the domestic
equilibrium price. American imports will increase when world prices fall relative to the
domestic price.

For Canada, there will be domestic supply and demand and export supply and import
demand for aluminum. The description of these supply and demand curves is similar for
those of the United States. The price and quantity of aluminum are determined by the
intersection of the domestic demand and supply curves in a world without trade. In a
world with trade, the export supply curve for Canada shows the amount of aluminum
that the Canadian producers will export at each world price above the domestic
equilibrium price. Canadian exports will increase when the world price rises relative to
the domestic price. The import demand curve for Canada shows the amount of
aluminum that Canadians will import at each world price below the domestic
equilibrium price. Canadian imports will increase when world prices fall relative to the
domestic price.

The equilibrium world price and equilibrium world levels of exports and imports of
aluminum can be determined with further supply and demand analysis. The export
supply curves of the United States and Canada can be plotted on one graph. The import
demand curves of both nations can be plotted on the same graph. In this two-nation
model, equilibrium will be achieved when the United States’ import demand curve for
aluminum intersects the Canadian export supply curve. [text: E pp. 737-741; MA pp.
381-385; MI pp. 479-483]

227
Technology, R&D, and Efficiency

13. Define the four basic types of trade barriers.

First, tariffs are excise taxes on imports. They may be used to collect revenue or
government for they may be protective tariffs that are supposed to protect domestic
producers from foreign competition. Second, import quotas specify the maximum
amounts of imports allowed into a nation over a period of time. Third, nontariff barriers
refer to licensing requirements, unreasonable standards, or bureaucratic red tape in
customs procedures. Fourth, there can be voluntary export restrictions, which are
agreements by foreign firms to “voluntarily” limit their exports to another nation. [text:
E p. 741; MA p. 385; MI p. 483]

14. Which is more effective in blocking imports, a tariff or a quota?

Generally, an import quota, especially if it is set low, is more effective in blocking the
entry imports into a nation. The reason is that once the import quota has been met, no
more goods can be imported into the nation. With a tariff, it is still possible to import
goods into a nation, so long as people are willing to pay the tariff on the imported good.
Of course the precise answer depends on how low the quota is and how high the tariff is
on the product. [text: E pp. 741-743; MA pp. 385-387; MI pp. 483-485]

228
Technology, R&D, and Efficiency

15. Who gains and who loses from a protective tariff? Explain.

A tariff increases the price of the imported product. First, consumers are hurt. Some
consumers will not be able to purchase the product at the higher price and the consumers
who do purchase the good will pay the higher price. Second, a tariff helps domestic
producers because they receive more revenue than they would without the tariff. Third,
workers in the tariff-protected industry may benefit because it may protect jobs and
maintain higher wages than would be the case without the tariff. Fourth, business and
workers in industries that import or service the import product are hurt. Fifth, foreign
producers of the imported product receive less revenue than would be the case with free
trade. Sixth, the federal government gains from a tariff because it receives the amount
of the tariff times the number of the products that are imported. Overall, the nation loses
because the gains for the industry protected by a tariff and workers in that industry are
significantly offset by the losses to consumers and economic inefficiency that comes
from an industry that cannot compete on world markets. [text: E pp. 741-742; MA pp.
385-386; MI pp. 483-485]

16. What are the similarities and differences in the economic effects of tariffs and quotas?

Tariffs and quotas have essentially the same economic effects. They both cause a
decline in consumption because of the higher price that results. They both increase
domestic production. They both reduce the quantity of imports. They also both have
the same indirect effects of promoting the expansion of inefficient industries that do not
have a comparative advantage. The major difference between a tariff and a quota is that
a tariff raises revenue for the government while a quota transfers revenue to foreign
producers. [text: E pp. 742-743; MA pp. 386-387; MI pp. 485-486]

17. “Unless a tariff is prohibitive, it does not inhibit competition as much as a quota.”
Evaluate.

A tariff raises the price of an import, but it does not limit the quantity imported. It is
also possible that the price of the import will not rise by the full extent of the tariff
imposed. The foreign manufacturer can absorb part or all of the tariff amount, or may
be motivated to find lower-cost production methods to reduce the price. Clearly, when
this happens the competition remains. Even where the price of the product is raised by
the entire amount of the tariff, competition remains and domestic producers are limited
in their own pricing decisions by the foreign competition.

A quota, however, limits the amount of the imported product to a specific amount. This
may encourage the foreign producer to raise prices to cover the loss in revenue that
resulted from the limited quantity. It will also cause a leftward shift in the total supply
(domestic plus foreign) curve to consumers, allowing domestic producers to also charge
a higher price. The end result of a quota is certain higher prices and lower quantities,
and no resulting tariff revenue for the Federal government. The end result of a tariff is
uncertain with regard to price, the quantity purchased may not decline, and the
government receives some tariff revenues. [text: E pp. 742-743; MA pp. 386-387; MI
pp. 485-486]

18. Do protectionist policies benefit producers, consumers, workers, or the government?


Explain.

Protectionism in the form of tariffs or quotas reduces the total supply (domestic plus
foreign) of the good. As a consequence, the price will rise. This change has a number
of profound effects on all groups. Some consumers will not be able to purchase the
product at the higher price. Those consumers that still purchase the good will pay a
higher price. So consumers are hurt by protectionist legislation. Domestic producers

229
Technology, R&D, and Efficiency

will receive more revenue and workers in those industries may benefit from the
improved revenue for business because it may mean fewer layoffs or higher wages.
Thus, domestic producers and workers in those industries benefit, which explains why
their organizations often lobby hard for the protectionist legislation. Businesses and
workers in industries that import or service foreign products are hurt by tariffs and
quotas. Foreign producers receive less revenue than would be the case with free trade.
The government will benefit from a tariff, but will receive no revenue from a quota.
Overall, the gains for protected industries and workers come at the expense of the whole
economy. [text: E pp. 741-743; MA pp. 385-387; MI pp. 483-486]

25. Explain four problems with the argument that trade protection is needed to protect
American jobs.

There are several problems with using trade protection to “save American jobs.” First,
imports may eliminate some jobs, but they create others in those industries that import
products. Second, there is a fallacy of composition. The imports of one nation are the
exports of another nation. Using trade policy to protect domestic jobs in the United
States will weaken the trading partners of the United States. The reason for this
weakness is that the trading partners will export less and thus have less income with
which to buy imports from the United States. Third, there is the possibility of retaliation
from trading partners that make all nations worse off. This problem occurred in the
1930s when high tariffs were imposed by the U.S. Smoot-Hawley Tariff Act of 1930.
Fourth, there are long-run feedback effects from an excess of exports over imports. This
policy leads to less income abroad that nations have to buy our goods. Workers in those
export industries in the United States are thus hurt and resources are reallocated to
protected industries at a great cost to the nation. [text: E p. 744; MA p. 388; MI p. 487]

32. How can the United States compete successfully with relatively low-wage nations such
as India and China?

If those low wages are a reflection of a low level of productivity, the per unit cost of
Indian and Chinese products will not be any greater than the unit costs in the U.S. It is
not the level of wages alone that determines the cost of a product, but rather the wage
relative to the level of output per hour. Wages in the U.S. are high, but so is the level of
productivity, often keeping unit costs as low or lower than those in low-wage nations.
[text: E p. 746; MA p. 390; MI p. 489]

CHAPTER 21 Exchange Rates, the Balance of Payments,


and Trade Deficits

1. What happens in the foreign exchange market when there is a U.S. export transaction?

When U.S. companies export goods and services there is an increase in foreign demand
for U.S. dollars. This increased demand is met by an increased supply of foreign
currencies owned by U.S. banks and available to U.S. buyers because the foreign firms
must purchase U.S. dollars for a fee from a bank to settle their accounts. [text: E pp.
752-753; MA pp. 396-397; MI pp. 494-495]

3. What are the major components of the current account in the balance of payments?
How is the current account balance determined?

The current account shows the position of the United States in terms of trade in goods
and services. Two major components of this account are U.S. goods exports and
imports. The difference between these two components gives the balance of trade. Two
other components are U.S. exports and imports of services. The difference between

230
Technology, R&D, and Efficiency

these two components gives the services surplus or deficit, which is added to the balance
of trade to give the balance on goods and services. The fifth component is net
investment income. It shows excess of interest and dividend payments foreigners paid
to U.S. individuals or companies for the services of U.S. exported capital over what the
U.S. paid to foreign individuals or companies. The sixth item is net transfer payments.
This is the difference between U.S. public and private transfer payments to the rest of
the world and foreign private and public transfer payments to the United States. Adding
net investment income and net transfer payments to the balance on goods and services
gives the balance on the current account. [text: E pp. 753-755; MA pp. 397-399; MI pp.
495-497]

4. Explain the relationship between the current account and the capital account in the
balance of payments.

The current account basically shows the position of the United States in terms of trade in
goods and services with the rest of the world during a year. The capital account shows
the capital flows in the purchase or sale of real and financial assets during a year. The
two accounts are interrelated.

A current account surplus basically means that the value of American exports of goods
and services was greater than imports of goods and services. This difference in value is
financed by Americans lending money abroad or by the purchase of foreign assets.
Thus, a current account surplus in the United States is financed by a net capital outflow
from the capital account.

By contrast, a current account deficit basically means that the value of American exports
of goods and services was less than imports of goods and services. This difference in
value is financed by Americans borrowing money from abroad or by selling ownership
of American goods and services to foreigners. A current account deficit in the United
States is financed by a net capital inflow from the capital account. [text: E pp. 753-755;
MA pp. 397-399; MI pp. 495-497]
5. What is the official reserves account and how is it used in the balance of payments?

The official reserves are the foreign currencies owned by the central bank. These
reserves decrease when they are used to finance a net deficit on the combined current
and capital accounts. The reserves increase when a nation has a net surplus on its
current and capital accounts. The three components of the balance of payments—the
current account, the capital account, and the official reserves account—must equal zero.
[text: E pp. 755-756; MA pp. 399-400; MI pp. 497-498]

6. What is a balance of payments deficit? What is a balance of payments surplus?

A nation is said to have a balance of payments deficit when imbalances in the combined
current and capital accounts lead to a decrease in official reserves. A balance of
payments surplus arises when imbalances in the combined current and capital accounts
result in an increase in official reserves. [text: E p. 756; MA p. 400; MI p. 498]

7. If a nation’s balance of payments is always in balance, why isn’t it also always in


equilibrium?

Official bank reserves are drawn upon to settle net differences in current and capital
account balances so that the balance of payments is brought into balance. However, the
change in the status of official reserves represents a disequilibrium in the balance of
payments. So-called autonomous transactions did not balance, and official reserves
were needed to accommodate the difference. [text: E p. 756; MA p. 400; MI p. 498]

231
Technology, R&D, and Efficiency

10. The table below contains the international balance of payments data for the United
States in a past year. All figures are in billions. Compute with the appropriate sign (+
or –) and enter in the table the six missing items. What was the condition of the balance
of payments in the United States?

232
Technology, R&D, and Efficiency

Current account
(1) U.S. goods exports $+390
(2) U.S. goods imports –498
(3) Balance of trade _____
(4) U.S. exports of services +133
(5) U.S. imports of services –107
(6) Balance on services _____
(7) Balance on goods and services _____
(8) Net investment income +12
(9) Net transfers –22
(10) Balance on current account _____

Capital account
(11) Foreign purchases of assets in the U.S. +117
(12) U.S. purchases of assets abroad –59
(13) Balance on capital account _____
(14) Current and capital account balance _____
(15) Official reserves _____
$_____

Current account
(1) U.S. goods exports $+390
(2) U.S. goods imports –498
(3) Balance of trade –108
(4) U.S. exports of services +133
(5) U.S. imports of services –107
(6) Balance on services +23
(7) Balance on goods and services –82
(8) Net investment income +12
(9) Net transfers –22
(10) Balance on current account –92

Capital account
(11) Foreign purchases of assets in the U.S. +117
(12) U.S. purchases of assets abroad –59
(13) Balance on capital account +58
(14) Current and capital account balance –34
(15) Official reserves +34
$0

The United States had a balance of payment deficit of $34 billion. [text: E pp. 753-755;
MA pp. 397-399; MI pp. 495-497]

12. Explain how a nation might persistently import more than it exports and still maintain an
equilibrium in its balance of payments.

A nation could persistently import more than it exports and still maintain an equilibrium
in its balance of payments in several ways. It could have an offsetting surplus in
transactions of other items in the current account such as services. More probably, the
offsetting transactions would come through the capital account. For example, foreign
investors could have a net excess in the demand for dollars equal to the excess in supply
of dollars created by the trade deficit. The demand for dollars to invest in U.S. assets
could maintain equilibrium in the balance of payments for many years. [text: E pp. 753-
756; MA pp. 397-400; MI pp. 495-498]

233
Technology, R&D, and Efficiency

13. Suppose that Mexico devalues the peso. What objectives would prompt the
devaluation? Be specific.

The objective of a devaluation is to correct a persistent overvaluation of a country’s


currency. What this means in the case of the peso is that Mexico must have had a
persistent balance of payments deficit which could not be easily corrected with
offsetting transfers of official reserves. The persistence of the deficit means that
Mexicans are importing more than they are exporting, that foreign investment in Mexico
does not offset the current account deficit, and all of that means that Mexican goods and
assets must be overvalued relative to foreign goods and services and assets. To correct
this imbalance, Mexico can devalue its currency, thus making purchases of Mexican
products and assets less expensive in terms of foreign currency. That is, foreigners can
now get more pesos for their currency. By itself, this would increase the quantity
demanded of pesos, but it should also shift the demand curve for pesos as Mexican
goods are now less expensive. All of this should help correct the balance of payments
deficit. [text: E pp. 756, 759-760; MA pp. 400, 403-404; MI pp. 498, 501-502]

14. Explain how the dollar price of an imported good may change even though the foreign
production cost of that product remains unchanged.

The dollar price depends on two things: the price of that product in the foreign country
and the price of that country’s currency in terms of dollars. If the price of the product
remains the same in the country, the dollar price of the product may still rise because the
price of that country’s currency in dollar terms might rise. If the value of the dollar has
depreciated against that country’s currency, it will take more dollars to purchase the
product even though its domestic price is unchanged. [text: E pp. 757-758; MA pp.
401-402; MI pp. 499-500]

15. List and explain the major determinants of the demand for, and supply of, the money of
a foreign nation.

(a) Changes in tastes: If consumer preferences for the products of a foreign nation
change, then the demand for and supply of that country’s currency will change. If the
demand for British woolens increases, then the demand for British pounds will increase
causing the pound to appreciate. If the demand for Rolls Royces declines, then the
demand for British pounds will decrease causing the pound to depreciate. Conversely, if
British preferences regarding some other country’s products change, then the supply of
British pounds will change accordingly.
(b) Relative income changes: If a foreign nation’s income rises more rapidly than other
nations, then its expenditures on imports is likely to grow along with expenditures on
everything else. This will probably cause the currency to depreciate as other nations are
not growing as rapidly, and therefore, their demand for the nation’s currency is not
keeping pace with the change in supply. The reverse would be true if the nation’s
income grew more slowly than other nations.
(c) Relative price changes: If prices in one country rise relatively more than in another,
the currency in the higher-priced country is likely to depreciate as domestic consumers
seek less expensive foreign imports and increase the demand for foreign currency. At
the same time demand for domestic currency will fall since foreigners will find it less
attractive to buy the higher-priced country’s goods. This, too, adds to the depreciation
of the higher-priced country’s currency.
(d) Relative real interest rates: Higher relative real interest rates in one country will
cause an increase in demand for the currency of that country or an appreciation of that
country’s currency as foreign investors seek higher rates of return. The reverse would
be true of lower relative real interest rates.
(e) Speculation: If speculators believe that the value of a currency will depreciate in the
future (for whatever reason), then they will attempt to convert that currency to another.

234
Technology, R&D, and Efficiency

This will cause an increase in the supply of the currency, which will cause it to
depreciate without an offsetting increase in demand. Thus, in effect, the speculators
help to cause what they predict. [text: E pp. 758-759; MA pp. 402-403; MI pp. 500-
501]

16. Explain how the exchange rate gets determined in a flexible exchange rate system.

If the foreign exchange rate floats freely, the demand for and the supply of foreign
currency determine foreign exchange rates. The exchange rate for any foreign currency
is the rate at which the quantity of that currency demanded is equal to the quantity
supplied. A change in the demand for or the supply of foreign currency will cause a
change in the exchange rate for that currency. When there is an increase in the price
paid in dollars for a foreign currency, the dollar has depreciated and the foreign currency
has appreciated in value. Conversely, when there is a decrease in the price paid in
dollars for a foreign currency, the dollar has appreciated and the foreign currency has
depreciated in value.

The demand for and supply of a foreign currency can change for many reasons. These
shifts occur because of changes in tastes, relative incomes, relative price-levels, relative
interest rates, and speculation. [text: E pp. 756-759; MA pp. 400-403 MI pp. 498-501]

17. How are flexible exchange rates used to eliminate a balance of payments deficit or
surplus?

Flexible exchange rates can be used to eliminate a balance of payments deficit or


surplus. When a nation has a balance of payments deficit, foreign exchange rates will
increase, thus making foreign goods and services more expensive and decreasing
imports. These events will make a nation’s goods and services less expensive for
foreigners to buy, thus increasing exports. With a balance of payments surplus, the
exchange rates will increase, thus making foreign goods and services less expensive and
increasing imports. This situation makes a nation’s goods and services more expensive
for foreigners to buy, thus decreasing exports. [text: E pp. 759-760; MA 403-404; MI
pp. 501-502]

18. Describe the three major disadvantages of flexible exchange rates.

First, flexible exchange rates are subject to great volatility, and thus create uncertainty in
exchange rate markets. This uncertainty can adversely affect trade because it creates
more risks for those businesses that import and export goods and services. Second, a
fall in the international value of a nation’s currency will worsen the terms of trade for
that nation. A fall in the value of a nation’s currency will require that nation to export
more goods and services to finance the previous level of imports. Third, changes in
exchange rates create instability in the domestic economy. Depreciation or appreciation
of a nation’s currency affects the domestic economy. Inflationary pressures can arise
from a depreciation in the value of a nation’s currency because it increases the demand
for exports and increases the prices of goods imported. Appreciation of a nation’s
currency reduces exports and increases imports possibly creating unemployment in
export-related industries. [text: E pp. 760-761; MA pp. 404-405; MI pp. 502-503]

19. How does a fixed exchange rate system work? How can a nation maintain its fixed
exchange rate?

In the fixed exchange rate system a nation might fix (or “peg”) its exchange rate with
another nation. In this case, the governments of these nations must intervene in the
foreign exchange markets to prevent shortages and surpluses caused by shifts in demand
and supply for foreign currencies.

235
Technology, R&D, and Efficiency

One way for a nation to stabilize foreign exchange is for its government to sell its
reserves of a foreign currency in exchange for its own currency (or gold) when there is a
shortage of the foreign currency. Conversely, a government would buy a foreign
currency in exchange for its own currency (or gold) when there is a surplus of the
foreign currency.

Currency reserves, however, may be limited and inadequate for handling large and
persistent deficits or surpluses, so it may use other means to maintain fixed exchange
rates. First, a nation might adopt trade policies that discourage imports and encourage
exports. Second, a nation might impose exchange rate controls and rationing; but these
policies tend to distort trade, lead to government favoritism, restrict consumer choice,
and create black markets. A third way for a nation to stabilize foreign exchange rates is
to use monetary and fiscal policy to reduce its national income and price level, and raise
interest rates relative to other nations. These events would lead to a decrease in demand
for and increase in the supply of different foreign currencies. [text: E pp. 761-763; MA
pp. 405-407; MI pp. 503-505]

25. What was the gold standard?

The gold standard was a currency system of fixed exchange rates. There were three
conditions for the establishment and operation of the gold standard. First, a nation
defined the value of its money in terms of a quantity of gold. When all nations
expressed the value of currency in terms of an amount of gold, then all currencies could
be priced in terms of gold, and therefore had a fixed relationship with each other.
Second, a fixed relationship must be maintained between the stock of gold and the
domestic money supply. Imbalances in the relationship would lead to domestic
macroeconomic adjustments in real domestic output, employment, and the price level.
Third, gold must be freely exported or imported to adjust economies for changing
conditions. [text: E pp. 763-764; MA pp. 407-408; MI pp. 505-506]

236

You might also like